top of page

UPSC questions on Environemnt and Ecology

Get the contents of this page in PDF.  And much more.

UPSC 2011

Question : Biodiversity forms the basis for human existence in the following ways : 
 

1. Soil formation
2. Prevention of soil erosion
3. Recycling of waste
4. Pollination of crops

 

Select the correct answer using the codes given below:
 

A 1, 2 and 3 only

B 2, 3 and 4 only

C 1 and 4 only

D 1, 2, 3 and 4

 

Answer : D

 

Explanation : Biological diversity helps in the formation and maintenance of soil structure and the retention of moisture and nutrient levels. Trees on the other hand, lower the water table and remove deposited salt from the upper soil horizons.

 

 

 

Question : Which one of the following is not a site for in-situ method of conservation of flora? 

A Biosphere Reserve

B Botanical Garden

C National Park

D Wildlife Sanctuary

 

Answer : B

Explanation : In-situ conservation is on-site conservation or the conservation of genetic resources in natural populations of plant or animal species. It is the process of protecting an endangered plant or animal species in its natural habitat, either by protecting or cleaning up the habitat itself, or by defending the species from preadtors. Around 4% of the total geographical area of the country is used for in-situ conservation. The following methods are presently used for in-situ conservation - biosphere reserves, national parks and wild life sanctuaries. Botanical gardens are included under Ex-situ conservation. 

 

 

 

Question : Consider the following statements: 

1. Biodiversity is normally greater in the lower latitudes as compared to the higher latitudes.
2. Along the mountain gradients, biodiversity is normally greater in the lower altitudes as compared to the higher altitudes.

Which of the statements given above is/are correct?

A 1 only

B 2 only

C Both 1 and 2

D Neither 1 nor 2


Answer : C

Explanation : Biodiversity increases as we move from Poles to the Tropics. Thus localities at lower latitudes have more species than localities at higher latitudes.
Higher you go= colder atmosphere = less biodiversity.

 

 

 

 

Question : Three of the following criteria have contributed to the recognition of Western Ghats-Sri Lanka and Indo-Burma regions as hotspots of biodiversity: 

1. Species richness
2. Vegetation density
3. Endemism
4. Ethno-botanical importance
5. Threat perception
6. Adaptation of flora and fauna to warm and humid conditions

Which three of the above are correct criteria in this context?

 

A 1, 2 and 6

B 2, 4 and 6

C 1, 3 and 5

D 3, 4 and 6


Answer : C

Explanation : To qualify as a hotspot, a region must meet two strict criteria: 
1. It must contain at least 1,500 species of vascular plants (> 0.5 percent of the world’s total) as endemics (endemic = native to a particular habitat) 
2. It must have lost at least 70 percent of its original habitat. 

So we choose Species richness (1) as well as Endemism (3). Along with this Threat perception is necessary (5) to take, because it makes the base of this concept. 

Question : Two important rivers — one with its source in Jharkhand (and known by a different name in Odisha), and another, with its source in Odisha — merge at a place only a short distance from the coast of Bay of Bengal before flowing into the sea. This is an important site of wildlife and biodiversity and a protected area.

Which one of the following could be this?

A Bhitarkanika

B Chandipur-on-sea

C Gopalpur-on-sea

D Simlipal


Answer :  A

Explanation : Bhitarkanika is a unique habitat of Mangrove Forests criss-crossed with numerous creeks and mud flats located in Kendrapara district of Orissa. One of the largest Mangrove Eco systems in India, Bhitarkanika is home to diverse flora and fauna.The Bhitarkanika Mangroves cover an area of 650 km2 in the river delta of the Brahmani and Baitarani rivers.

 

 

 

 

Question : A sandy and saline area is the natural habitat of an Indian animal species. The animal has no predators in that area but its existence is threatened due to the destruction of its habitat. Which one of the following could be that animal? 


A Indian wild buffalo

B Indian wild ass

C Indian wild boar

D Indian gazelle


Answer :  B

Explanation : Indian wild ass is actually a threatened species available in the Rann of Kutch which is saline and sandy

 

 

 

 

Question : The Himalayan Range is Very rich in species diversity. Which one among the following is the most appropriate reason for this phenomenon?


A It has a high rainfall that supports luxuriant vegetative growth

B It is a confluence of different bio-geographical zones

C Exotic and invasive species have not been introduced in this region

D It has less human interference


Answer : B

Explanation : The Himalayan Range is very rich in species diversity because of the entire Himalayan region is very diverse bio-geographically. Hence option (B) - It is a confluence of different bio-geographical zones.

Question : In the Union Budget 2011-12, a full exemption from the basic customs duty was extended to the bio-based asphalt (bioasphalt). What is the importance of this material? 

1. Unlike traditional asphalt, bio-asphalt is not based on fossil fuels.
2. Bioasphalt can be made from non-renewable resources.
3. Bioasphalt can be made from organic waste materials.
4. It is eco-friendly to use bioasphalt for surfacing of the roads.

Which of the statements given above are correct?

A 1, 2 and 3 only

B 1, 3 and 4 only

C 2 and 4 only

D 1, 2, 3 and 4

 

Answer :

Explanation : Bioasphalt is an alternative to asphalt . It is derived from non-petroleum based renewable resources. which include sugar,molasses, rice ,natural tree, vegetable oils,cellulose,coconut waste.corn and potato starches and so on. In view of the drastic climatic changes and pollution non petroleum alternatives are preferred as they are more eco friendly.

It can only be made from renewable sources.  It is made from organic waste and is considered as eco-friendly.

Question : Consider the following:  

1. Carbon dioxide
2. Oxides of Nitrogen
3. Oxides of Sulphur

Which of the above is/are the emission/ emissions from coal combustion at thermal power plants?

A 1 only

B 2 and 3 only

C 1 and 3 only

D 1, 2 and 3


Answer : D

Explanation : The main emissions from coal combustion at thermal power plants are carbon dioxide (CO2), nitrogen oxides, sulfur oxides, chlorofluorocarbons (CFCs), and airborne inorganic particles such as fly ash and soot; CO2, methane, and CFCs are greenhouse gases.

 

 

 

 

Question : The formation of ozone hole in the Antarctic region has been a cause of concern. What could be the reason for the formation of this hole?

A Presence of prominent tropo-spheric turbulence; and inflow of chlorofluorocarbons

B Presence of prominent polar front and stratospheric’ clouds; and inflow of chlorofluorocarbons

C Absence of polar front and stratospheric clouds; and inflow of methane and chlorofluorocarbons

D Increased temperature at polar region due to global warming


Answer : B

Explanation : The severe depletion of the Antarctic ozone layer known as the “ozone hole” occurs because of the special atmospheric and chemical conditions that exist there and nowhere else on the globe. The very low winter temperatures in the Antarctic stratosphere cause polar stratospheric clouds (PSCs) to form. Ozone depletion occurs when chlorofluorocarbons (CFCs) and halons—gases (formerly found in aerosol spray cans and refrigerants) are released into the atmosphere. CFCs and halons cause chemical reactions that break down ozone molecules, reducing ozone's ultraviolet radiation-absorbing capacity

 

 

 

 

Question : Regarding “carbon, credits”, which one of the following statements is not correct?

A The carbon credit system was ratified in conjunction with the Kyoto Protocol

B Carbon credits are awarded to countries or groups that have reduced greenhouse gases below their emission quota

C The goal of the carbon credit system is to limit the increase of carbon dioxide emission

D Carbon credits are- traded at a price fixed from time to time by the United Nations Environment Programme


Answer :

Explanation : Carbon credits mean certificates granting the right to beholder to emit 1 tonne of carbon dioxide or its equivalent. These are generally awarded to groups or countries that have reduced greenhouse gases emissions below their quota. The carbon credit system was ratified in conjunction with Kyoto protocol and is aimed at reducing greenhouse gas emissions.

 

Question : There is a concern: over the increase in harmful algal blooms in the seawaters of India. What could be the causative factors for this phenomenon? 

1. Discharge of nutrients from the estuaries.
2. Run-off from the land during the monsoon.
3. Upwelling in the seas.

Select the correct answer from the codes given below:

A 1 only

B 1 and 2 only

C 2 and 3 only

D 1, 2 and 3


Answer : D

Explanation : An algal bloom is a rapid increase or accumulation in the population of algae (typically microscopic) in a water system. Cyanobacteria blooms are often called blue-green algae. Algal blooms may occur both in freshwater as well as marine environments. 
Typically, only one or a small number of phytoplankton species are involved, and some blooms may be recognized by discoloration of the water resulting from the high density of pigmented cell. Algal blooms are the result of an excess of nutrients, particularly some phosphates.The excess of nutrients may originate from fertilizers that are applied to land for agricultural or recreational purposes. They may also originate from household cleaning products containing phosphorus. These nutrients can then enter watersheds through water runoff. Excess carbon and nitrogen have also been suspected as causes. Presence of residual sodium carbonate acts as catalyst for the algae to bloom by providing dissolved carbon dioxide for enhanced photo synthesis in the presence of nutrients. 
When phosphates are introduced into water systems, higher concentrations cause increased growth of algae and plants. Algae tend to grow very quickly under high nutrient availability, but each alga is short-lived, and the result is a high concentration of dead organic matter which starts to decay. The decay process consumes dissolved oxygen in the water, resulting in hypoxic conditions. Without sufficient dissolved oxygen in the water, animals and plants may die off in large numbers.

Harmful algal bloom (HAB) is an algal bloom that causes negative impacts to other organisms via production of natural toxins, (or otherwise harmful phytoplankton such as dinoflagellates ) mechanical damage to other organisms, or by other means. Such blooms often take on a red or brown hue and are known as red tides. HABs are often associated with large-scale marine mortality events and have been associated with various types of shellfish poisonings too.

The frequency and severity of HABs in some parts of the world have been linked to increased nutrient loading from human activities. In other areas, HABs are a predictable seasonal occurrence resulting from coastal upwelling. The growth of marine phytoplankton (both non-toxic and toxic) is generally limited by the availability of nitrates and phosphates, which can be abundant in coastal upwelling zones as well as in agricultural run-off areas. The type of nitrates and phosphates available in the system are also a factor, since phytoplankton can grow at different rates depending on the relative abundance of these substances (e.g. ammonia, urea, nitrate ion).

A variety of other nutrient sources can also play an important role in affecting algal bloom formation, including iron, silica or carbon. Coastal water pollution produced by humans (including iron fertilization) and systematic increase in sea water temperature have also been suggested as possible contributing factors in HABs. Other factors such as iron-rich dust influx from large desert areas such as the Sahara are thought to play a role in causing HABs.

Climate Change can also cause Algal boom. Coastal upwelling is the process by which winds push surface water offshore and deep water moves towards the coast, bringing nutrients from the ocean floor to the surface. Climate change is expected to alter the timing and intensity of coastal upwelling. Along the west coast of the United States, excess nutrients delivered by upwelling might lead to more algal blooms.
Some algal blooms on the Pacific coast have also been linked to natural occurrences of large-scale climatic oscillations such as El Niño events.
Climate change might lead to more droughts, which make freshwater saltier. This can cause marine algae to invade freshwater ecosystems.
Algae need carbon dioxide to survive. Higher levels of carbon dioxide in the air and water can lead to rapid growth of algae, especially toxic blue-green algae that can float to the surface of the water.
Climate change might affect rainfall patterns, leading to alternating periods of drought and intense storms. This can cause more nutrient runoff into water bodies, feeding more algal blooms.
Scientists predict that sea level could rise up to one meter by the year 2100. This would create more shallow and stable coastal water, conditions that are perfect for the growth of algae

Warmer water due to climate change might favor harmful algae in a number of ways:

1.    Toxic blue-green algae prefer warmer water.
2.    Warmer temperatures prevent water from mixing, allowing algae to grow thicker and faster.
3.    Warmer water is easier for small organisms to move through and allows algae to float to the surface faster.
4.    Algal blooms absorb sunlight, making water even warmer and promoting more blooms.

 

 

 

 

Question : The “Red Data Books” published by the International Union for Conservation of Nature and Natural Resources (IUCN) contain lists of:

1. Endemic plant and animal species present in the biodiversity hotspots,
2. Threatened plant and animal species.
3. Protected sites for conservation of nature and natural resources in various countries.

Select the correct answer using the codes given below:

A 1 and 3

B 2 only

C 2 and 3

D 3 only

 

Answer :  B

Explanation : Red data book is the document established by IUCN for documenting the rare and endangered species of plants, animals, fungi and also a few local species that exist within a state or country.

 

 

Question : Human activities in the recent past have Caused the increased concentration of carbon dioxide in the atmosphere, but a lot of it does not remain in the lower atmosphere because of : 

1. its escape into the outer stratosphere.
2. the photosynthesis by phyto-plankton in the oceans.
3. the trapping of air in the polar ice caps.

Which of the statements given above is/are correct?

A 1 and 2

B 2 only

C 2 and 3

D 3 only


Answer :  C

Explanation : Phytoplankton are a very important part of the ocean foodchain. They make their own food by photosynthesis for which they utilize atmospheric carbon dioxide.

Climate change is a fact, and most of the warming is caused by human activity. The Arctic is now so warm that the extent of sea ice has decreased by about 30 percent in summer and in winter, sea ice is getting thinner.
New research shows that sea ice removes carbon dioxide from the atmosphere. If Arctic sea ice is reduced, we may therefore be facing an increase of atmospheric concentration of carbon dioxide

 

Question : In the context of ecosystem productivity, marine upwelling zones are important as they increase the marine productivity by bringing the:

1. decomposer microorganisms to the surface.
2. nutrients to the surface.
3. bottom-dwelling organisms to- the surface.

Which of the statements given above is/are correct?

A 1 and 2

B 2 only

C 2 and 3

D 3 only


Answer : C

 

Explanation : Upwelling brings nutrient -rich water towards the ocean surface replacing the warmer and usually nutrient depleted surface water. This is to do with the Phytoplankton and nothing to do with the decomposer microorganisms

 

Question : With reference to India, consider the following Central Acts: 

1. Import and Export (Control) Act, 1947
2. Mining and Mineral Development (Regulation) Act, 1957
3. Customs Act, 1962
4. Indian Forest Act, 1927

Which of the above Acts have relevance to/bearing on the biodiversity conservation in the country?

A 1 and 3 only

B 2, 3 and 4 only

C 1, 2, 3 and 4

D None of the above Acts

 

Answer : C

Explanation : All four Acts are related to biodiversity. Import and export of GMOs or exotic species is prohibited whereas certain medicial plants are subjected to high custom duties to regulate their trade. Minus regulation and forest conservation anyway involve provisions related to biodiversity conservation

APSC Test Series

Attempt an exclusive paper on Environment and Ecology in our APSC GS Prelims Test Series.


Click here to register

UPSC 2012

Question: Consider the following kinds of organisms :

 

1. Bacteria
2. Fungi
3. Flowering plants

Some species of which of the above kinds of organisms are employed as biopesticides?

A 1 only

B 2 and 3 only

C 1 and 3 only

D 1, 2 and 3

Answer : D

Explanation : Biopesticides are certain types of pesticides derived from such natural materials as animals, plants, bacteria, and certain minerals. For example, canola oil and baking soda have pesticidal applications and are considered biopesticides. Viruses, bacteria, fungi, protozoa, mites, insects, and flowers have all been used
as biopesticides.

Question: Biomass gasification is considered to be one of the sustainable solutions to the power crisis in India. In this context, which of the following statements is/are
correct? 

1. Coconut shells, groundnut shells and rice husk can be used in biomass gasification.
2. The combustible gases generated from biomass gasification consist of hydrogen and carbon dioxide only.
3. The combustible gases generated from biomass gasification can be used for direct heat generation but not in internal combustion engines.

Select the correct answer using the codes given below :

A 1 only

B 2 and 3 only

C 1 and 3 only

D 1, 2 and 3

 

Answer : A

Explanation : Biomass gasification is a new technology in which biomass (renewable organic resource like agriculture crop residues, forest residues, special crops grown particularly for energy use, organic  municipal solid waste and animal waste) is converted to hydrogen and other products without combustion using heat, steam and oxygen. As such Statement (1) Coconut shells, groundnut shells and rice husk can be used in biomass gasification is correct. Statement (2) is incorrect because of the use of the word "only".
Statement (3) is incorrect because some new IC engines are using biomass gasification 

Question: Consider the following areas:

1. Bandipur
2. Bhitarkanika
3. Manas
4. Sunderbans

Which of the above are Tiger Reserves?

A 1 and 2 only

B 1, 3 and 4 only

C 2, 3 and 4 only

D l, 2, 3 and 4

 

Answer : B

Explanation : Bandipur National Park, a tiger reserve is located in the south Indian state of Karnataka. 
Manas National Park or Manas Wildlife Sanctuary is a National Park, UNESCO Natural World Heritage site, a Project Tiger Reserve, an Elephant Reserve and a Biosphere Reserve in Assam. 
The Sundarban National Park is a National Park, Tiger Reserve, and a Biosphere Reserve in India. It is a part of the Sundarbans on the Ganges Delta of India and Bangladesh. 
Bhitarkanika National Park is a national park located in the Kendrapara District Odisha, which is not specifically for Tiger reserve

 

Question: In which one among the following categories of protected areas in India are local people not allowed to collect and use the biomass?

A Biosphere Reserves

B National Parks

C Wetlands declared under Ramsar Convention

D Wildlife Sanctuaries
 

Answer : B

Explanation : In India, local people are not allowed to collect and use the biomass in National Parks.
This is so because National Parks are state property and are only used by the government. They are not for public use. These parks are only used for tourism and reservation purposes.

 

Question: Consider the following kinds of organisms :

1. Bat
2. Bee
3. Bird

Which of the above is/are pollinating agent/agents?

A 1 and 2 only

B 2 only

C 1 and 3 only

D 1, 2 and 3

Answer : D

Explanation : Bees, bats and birds are the biotic pollinating agents. The majority of pollinators are biotic agents such as insects (like bees, flies, and butterflies), bats, birds, and other animals. Other plant species are pollinated by abiotic agents, such as wind and water.

 

Question: Which one of the following groups of animals belongs to the category of endangered species?

A Great Indian Bustard, Musk Deer, Red Panda and Asiatic Wild Ass

B Kashmir Stag, Cheetal, Blue Bull and Great Indian Bustard

C Snow Leopard, Swamp Deer, Rhesus Monkey and Saras (Crane)

D Lion-tailed Macaque, Blue Bull, Hanuman Langur and Cheetal

Answer : A

Explanation : Option (A) has the list of all endangered species 

Elimination of other options . 
(B) Kashmir Stag is not endangered 
(C) Saras (crane) is not endangered
(D) Cheetal (spotted deer) is not endangered

 

 

Question: Which of the following can be threats to the biodiversity of a geographical area?

1. Global warming
2. Fragmentation of habitat
3. Invasion of alien species
4. Promotion of vegetarianism

Select the correct answer using the codes given below :

A 1, 2 and 3 only

B 2 and 3 only

C 1 and 4 only

D 1, 2, 3 and 4

Answer : A

Explanation : Refer to the PDF here - http://www.indiaenvironmentportal.org.in/files/National_Biodiversity_Action_Plan.pdf
From the table of contents, you can see that Global warming, fragmentation of habitat and invasion of alien species are a threat.

 

Question: Consider the following :

 

1. Black-necked crane
2. Cheetah
3. Flying squirrel
4. Snow leopard

Which of the above are naturally found in India?


A 1, 2 and 3 only

B 1, 3 and 4 only

C 2 and 4 only

D 1, 2, 3 and 4

Answer : D

Explanation : Black-necked crane is commonly found in Tibetan and trans-Himalayan region. In winters they migrate to less colder regions of Indian Himalayas.
Cheetah is an extinct species. They have gone extinct during pre-independence era due to the fact that they were hunted down by various Indian kings and British officers.
Flying Squirrels are found in many Indian forests.
Snow leopard is an ‘endangered’ specie found in the Himalayan ranges.

 

Question: A particular State in India has the following characteristics :

It is located on the same latitude which passes through northern Rajasthan.
It has over 80% of its area under forest cover.
Over 12% of forest cover constitutes Protected Area Network in this State.

Which one among the following States has all the above characteristics?

A Arunachal Pradesh

B Assam

C Himachal Pradesh

D Uttarakhand

Answer : A

Explanation : Himachal Pradesh is located north of Rajasthan, so it can't be on the same latitude.
Only in North East and Andaman can 80% of area be under forests and so Himachal and Uttarakhand ruled out. Forest area in Assam can not be >80% since a  large part of Assam is valley. It’s forest area is actually <50%
12% of the total area of Arunachal Pradesh and 15% of the total forest cover in being protected by the Department of Environment and Forests of Arunachal Pradesh.

 

 

Question: With reference to the wetlands of India, consider the following statements :

1. The country’s total geographical area under the category of wetlands is recorded more in Gujarat as compared to other States.
2. In India, the total geographical area of coastal wetlands larger than that of wetlands.

Which of the statements given above is/are correct?

A 1 only

B 2 only

C Both 1 and 2

D Neither 1 nor 2

Answer : A

Explanation : Wetlands cover around 3% of the total land area of India. Gujarat occupies around 3 million hectares of wetlands out of India’s total 10 million hectares of wetland area. India has total 27,403 wetlands, of which 23,444 are inland wetlands and 3,959 are coastal wetlands. However area wise the coastal wetlands occupy an estimated 6,750 sq km, which is more than the inland wetlands.

 

 

Question: The acidification of oceans is increasing. Why is this phenomenon a cause of concern? 

1. The growth and survival of calcareous phytoplankton will be adversely affected.
2. The growth and survival of coral reefs will be adversely affected.
3. The survival of some animals that have phytoplanktonic larvae will be adversely affected.
4. The cloud seeding and formation of clouds will be adversely affected.

Which of the statements given above is /are correct?

A 1, 2 and 3 only

B 2 only

C 1 and 3 only

D 1, 2, 3 and 4

Answer : D

Explanation : 2 is correct since corals have calcium Skelton, acid will dissolve that skeleton
1 is also true with similar logic, calcareous phytoplankton have calcium Skelton will be dissolved by acid
4 Marine phytoplankton releases sulphur compounds into the atmosphere that contribute to cooling the planet. But ocean acidification could hinder this process.
Acidification would lead certain marine organisms to emit less of the sulphur compounds that help to seed the formation of clouds and so keep the planet cool.
Statements 1,2 and 4 correct and answer is Option D

 

Question: The Millennium Ecosystem Assessment describes the following major categories of ecosystem services-provisioning, supporting, regulating, preserving and cultural.

 

Which one of the following is supporting service? 

A Production of food and water

B Control of climate and disease

C Nutrient cycling and crop pollination

D Maintenance of diversity

Answer : C

Explanation : Production of food and water comes into the category of Provisioning services of ecosystem.

Control of climate and disease is in the category of Regulating services of ecosystem.

Nutrient cycling and pollination is in the category of Supporting services.
These services are essential for all other services and for basic sustenance of ecosystem.

Maintenance of diversity is in the category of cultural services. These are non material features and benefits of ecosystem.

Additional read - https://en.wikipedia.org/wiki/Ecosystem_service#Supporting_services  (under categorization)

 

 

Question: What would happen if phytoplankton of an ocean is completely destroyed for some reason? 

1. The ocean as a carbon sink would be adversely affected.
2. The food chains in the ocean would be adversely affected.
3. The density of ocean water would drastically decrease.

Select the correct answer using the codes given below :

A 1 and 2 only

B 2 only

C 3 only

D 1, 2 and 3

Answer : A

Explanation : Phytoplankton which live on the ocean surfaces are marine algae, which get their nutrients through photosynthesis. In the process, they use up carbon dioxide and release oxygen. They are the most important source of food for higher order animals. So Statement 1 and 2 are correct

Statement 3 - The use of the word drastically is questionable . Oceanographers have not previously considered plankton as contributors to ocean density gradients, which are conventionally calculated from temperature and salinity alone in the upper ocean. However they found that phytoplankton density gradients were in some cases comparable to the physical density gradients. As small density gradients drive some ocean currents, and help control gas and heat exchange between ocean and
atmosphere, plankton are considered undescribed variable influencing these processes.

 

Therefore correct answer is A.

 

 

Question: What is the role of ultraviolet (UV) radiation in the water purification systems?

1. It inactivates /kills the harmful microorganisms in water.
2. It removes all the undesirable odours from the water.
3. It quickens the sedimentation of solid particles, removes turbidity and improves the clarity of water.

Which of the statements given above is/are correct?

A 1 only

B 2 and 3 only

C 1 and 3 only

D 1, 2 and 3

Answer : A

Explanation : A UV radiation is water purifier treats micro-biologically unsafe water with germicidal ultraviolet light. The UV wavelength scrambles the DNA of living organisms in the water, so they can no longer reproduce and make you sick. If you drink bacteria-infested water, the organisms can embed in your digestive tract and replicate.

Hence Option A is correct

 

Question: Consider the following statements : Chlorofluorocarbons, known as ozone-depleting substances, are used 

1. in the production of plastic foams
2. in the production of tubeless tyres
3. in cleaning certain electronic components
4. as pressurizing agents in aerosol cans

Which of the statements given above is/are correct?

A 1, 2 and 3 only

B 4 only

C 1, 3 and 4 only

D 1, 2, 3 and 4

Answer : C

Explanation : Chlorofluorocarbons or CFCs were used  - 
-as pressurizing agents in aerosol cans, 
-for rigid foam insulation such as polyurethane is produced by injecting CFCs into a liquid mass of plastic polymer, thereby creating the bubbles that provide insulating capabilities
-for cleaning electronic equipment.

 

 

Question: Government of India encourages the cultivation of ‘sea buckthorn’. What is the importance of this plant? 
 

1. It helps in controlling soil erosion and in preventing desertification.
2. It is a rich source of biodiesel.
3. It has nutritional value and is well-adapted to live in cold areas of high altitudes.
4. Its timber is of great commercial value.

Which of the statements given above is /are correct?

A 1 only

B 2, 3 and 4 only

C 1 and 3 only

D 1, 2, 3 and 4

 

Answer : C
 

Explanation : Government of India encourages the cultivation of sea buckthorn the importance of this plant is it helps in controlling soil erosion and its prevent desertification also it has a nutritional value and well adapted to live in cold regions.

APSC Test Series

Attempt an exclusive paper on Environment and Ecology in our APSC GS Prelims Test Series.


Click here to register

UPSC 2013

Question : Which of the following can be found as pollutants in the drinking water in some parts of India? 

1. Arsenic
2. Sorbitol
3. Fluoride
4. Formaldehyde
5. Uranium

Select the correct answer using the codes given below.

A 1 and 3 only

B 2, 4 and 5 only

C 1, 3 and 5 only

D 1, 2, 3, 4 and 5

Answer : C


Explanation : please refer to https://www.wqa.org/learn-about-water/common-contaminants for the list of common pollutants
Sorbitol (Item 2) used as artificial sweetener and laxative. Formaldehyde (Item
 4) used in coating, laminating etc. Neither of them not as famous water pollutants as arsenic and fluoride.

 

 

Question : In the grasslands, trees do not replace the grasses as a part of an ecological succession because of : 

A insects and fungi

B limited sunlight and paucity of nutrients

C water limits and fire

D None of the above

Answer : C

Explanation : Ecological succession is the process that describes how the structure of a biological community (that is, an interacting group of various species in a desert, forest, grassland, marine environment, and so on) changes over time. The two main types are primary succession and secondary succession

In primary succession, newly exposed or newly formed rock is colonized by living things for the first time. 
In secondary succession, an area that was previously occupied by living things is disturbed, then re-colonized following the disturbance.

The stages of primary succession include pioneer microorganisms, plants (lichens and mosses), grassy stage, smaller shrubs, and trees. Animals begin to return when there is food there for them to eat. When it is a fully functioning ecosystem, it has reached the climax community stage. That means trees eventually replace the grass or the grassy stage. 

However in grasslands the trees do no replace the grasses because 


Grass ecosystems are an early stage of succession in regions where the mature ecosystems are forests. However, grass ecosystems are climax ecosystems in grassland regions, where there is not enough rainfall to support a forest. (e.g. in our option C water limits and forest fire) (ref. Page 149 Human Ecology: Basic Concepts for Sustainable Development By Gerald G G Marten) . Ecological succession of a grassland is desert. (Page 3.17 Environmental Studies By Anjali Bagad)

Two factors for evolution of temperate grassland biome, 
i) Extreme continental climate with limited supply of water due to low rainfall 

ii) human activities mainly burning of vegetation.(Ref. Page 619, Physical Geography by Savindra Singh)
 

Therefore option C is correct

 

 

 

Question : Which one of the following is the correct sequence of ecosystems in the order of decreasing productivity? 

A Oceans, lakes, grasslands, mangroves

B Mangroves, oceans, grasslands, lakes

C Mangroves, grasslands, lakes, oceans

D Oceans, mangroves, lakes, grasslands

Answer : C

Explanation : This is a tricky question and as per official UPSC key, the answer is C.

Here by productivity they are referring to the rate of organic matter or biomass production. The productivity of an ecosystem is of two types: primary and secondary.

According to the source - https://ci.coastal.edu/~sgilman/770productivitynutrients.htm

- The average ocean productivity is about 50 grams carbon per square meter per year. The productivity of the open ocean (ocean away from coasts) is comparable to desert production. This means that most of the ocean, or about 90%, is essentially desert.

- Average land productivity is 160 grams carbon per square meter per year.

Hence Ocean must be the least productive and the only option confirming this is Option C

 

 

Question : Acid rain is caused by the pollution of environment by 

A carbon dioxide and nitrogen

B carbon monoxide and carbon dioxide

C ozone and carbon dioxide

D nitrous oxide and sulphur dioxide

Answer : D

Explanation : Acid rain is caused by a chemical reaction that begins when compounds like sulfur dioxide and nitrous oxides are released into the air. These substances can rise very high into the atmosphere, where they mix and react with water, oxygen, and other chemicals to form more acidic pollutants, known as acid rain.

 

 

Question : With reference to food chains in ecosystems, consider the following statements : 

1. A food chain illustrates the order in which a chain of organisms feed upon each other.
2. Food chains are found within the populations of a species.
3. A food chain illustrates the numbers of each organism which are eaten by others.

Which of the statements given above is / are correct?


A 1 only

B 1 and 2 only

C 1, 2 and 3

D None

 

Answer : A

Explanation : By definition - A food chain is a linear sequence of organisms through which nutrients and energy pass as one organism eats another.

1 is correct because food chain shows the sequence or order in which organism feed

2 is incorrect because food chain means passing of energy from one species to another. Thus cannibalism will stop a food chain and it will not be found within the populations of the species

3. is incorrect because food chain doesn't show the numbers but merely the flow of energy

Therefore only option A is correct

 

 

Question : Consider the following organisms: 

1. Agaricus
2. Nostoc
3. Spirogyra

Which of the above is / are used as biofertilizer / biofertilizers

A 1 and 2

B 2 only

C 2 and 3

D 3 only

Answer : B

Explanation : Agaricus is a genus of mushrooms containing both edible and poisonous species, not an bio fertilizer.

Nostoc fixes atmospheric nitrogen into ammonia, which may then be used or converted to a form suitable for plant growth. Thus Nostoc is a sustainable biofertilizer.

The algae grow on their own in the rice fields, but all of them are not beneficial to crop. The green algae like Srirogyra compete with the crop for nutrient and reduce tilering in the crop. Therefore it is not a biofertilizer.

 

 

Question : Which one of the following terms describes not only the physical space occupied by an organism, but also its functional role in the community of organisms? 


A Ecotone

B Ecological niche

C Habitat

D Home range

Answer : B

Explanation : Ecological niche is a term for the position of a species within an ecosystem, describing both the range of conditions necessary for persistence of the species, and its ecological role in the ecosystem.

The ecological niche describes how a species interacts within an ecosystem. The niche of a species depends on both biotic and abiotic factors, which affect the ability of a species to survive and endure.

Biotic factors affecting a species' niche include food availability and predators. Abiotic factors affecting ecological niche include temperature, landscape characteristics, soil nutrients, light and other non-living factors.

An example of an ecological niche is that of the dung beetle. The dung beetle, as its name suggests, consumes dung both in larval and adult form. Dung beetles store dung balls in burrows, and females lay eggs within them.

This allows hatched larvae immediate access to food. The dung beetle in turn influences the surrounding environment by aerating soil and rereleasing beneficial nutrients. Therefore, the dung beetle performs a unique role in its environment.

 

Question : Photochemical smog is a resultant of the reaction among: 

A NO2, 03 and peroxyacetyl nitrate in the presence of sunlight

B CO, 02 and peroxyacetyl nitrate in the presence of sunlight

C CO, CO2 and N02 at low temperature

D High concentration of N02, O3 and CO in the evening

Answer : A

Explanation : Photochemical smog is a type of smog produced when ultraviolet light from the sun reacts with nitrogen oxides of the atmosphere. 
To begin the chemical process of photochemical smog development the following conditions must occur.
1. sunlight
2. the production of oxides of nitrogen (NOx)
3. the production of volatile organic compounds
4. the temperature greater than 18 degrees celsius. 
reactions :
O3 + NO  ------------>   NO2 + O2
NO + RO2 ---------------->  NO2 + other products
NO2 + sunlight ----------->  NO + O
O + O2 ------------> O3
NO2 + R  --------------------> peroxyacetyl nitrate.

 

Question : With reference to the food chains in ecosystem, which of the following kinds of organism is / are known as decomposer organism/organisms? 

1. Virus
2. Fungi
3. Bacteria

Select the correct answer using the codes given below.

A 1 only

B 2 and 3 only

C 1 and 3 only

D 1, 2 and 3


Answer : B

Explanation : Decomposers are organisms that break down dead or decaying organisms. Bacteria and Fungi are considered as decomposer organisms. Viruses invade other organisms, but they're not decomposers.
Bacteria are important decomposers; they can break down any kind of organic matters. Fungi are primary decomposers, they not only decompose the surface organisms but they can also penetrate deep into the organic matters.

APSC Test Series

Attempt an exclusive paper on Environment and Ecology in our APSC GS Prelims Test Series.


Click here to register

UPSC 2014

Question : Which one of the following is the process involved in photosynthesis?
 

A. Potential energy is released to form free energy
B. Free energy is converted into potential energy and stored
C. Food is oxidized to release carbon dioxide and water
D. Oxygen is taken, and carbon dioxide and water vapour are given out

Answer : B

Explanation : Photosynthesis is the process in which plants make food using co2 and sunlight  and the process also releases oxygen . Since energy is being stored in the form of food, the process converts free energy (of the sun) into potential energy 

 

 

Question : Which of the following adds/add carbon dioxide to the carbon cycle on the planet Earth?
 

1. Volcanic action
2. Respiration
3. Photosynthesis
4. Decay of organic matter

Select the correct answer using the code given below.

A. 1 and 3 only
B. 2 only
C. 1, 2 and 4 only
D. 1, 2, 3 and 4

Answer : C

Explanation : Statement 1 - Volcanoes emit carbon dioxide during eruptions and also through magma which flows underground
Statement 2 - Co2 is released during respiration and it is a common knowledge
Statement 3 - plants make food using co2 and sunlight  and the process also releases oxygen. Since CO2 is absorbed from the atmosphere during photosynthesis, it is not released
Statement 4 - Organic matter decay releases carbon back into the atmosphere as CO2

 

Question : In India, the problem of soil erosion is associated with which of the following?

1. Terrace cultivation
2. Deforestation
3. Tropical climate

Select the correct answer using the code given below.

A. 1 and 2 only
B. 2 only
C. 1 and 3 only
D. 1, 2 and 3

Answer : B

Explanation : Statement 1 - terrace farming prevents soil erosion as it binds the soil together through food cultivation in hilly areas
Statement 2 - Deforestation leading to soil erosion is a common knowledge
Statement 3 - Tropical climate with heavy rainfall can lead to soil erosion.

Thus Answer is B 

 

Question : Lichens, which are capable of initiating ecological succession even on a bare rock, are actually a symbiotic association of

A. algae and bacteria
B. algae and fungi
C. bacteria and fungi
D. fungi and mosses

Answer : B

Explanation : Lichen is actually a stable symbiotic association between a fungus and algae and/or cyanobacteria. The carbon required by the lichen is provided by algae or cyanobacteria, which are photosynthetic

 

Question : The most important strategy for the conservation of biodiversity together with traditional human life is the establishment of 

A. biosphere reserves
B. botanical gardens
C. national parks
D. wildlife sanctuaries

Answer : A

Explanation : Biosphere reserves include terrestrial, marine and coastal ecosystems. Each site promotes solutions reconciling the conservation of biodiversity with its sustainable use. Biosphere reserves are an ecosystem with plants and animals of unusual scientific and natural interest.

Their main objective is to conserve diversity and integrity of plants animals and microorganisms.

 

Question : With reference to ‘Eco-Sensitive Zones’, which of the following statements is/are correct?

1. Eco-Sensitive Zones are the areas that are declared under the Wildlife (Protection) Act, 1972.
2. The purpose of the declaration of Eco-Sensitive Zones is to prohibit all kinds of human activities, in those zones except agriculture.

Select the correct answer using the code given below.

A. 1 only
B. 2 only
C. Both 1 and 2
D. Neither 1 nor 2

Answer : D

Explanation : Eco sensitive zones are declared under Environment (Protection) Act, 1986 (not under Wildlife (Protection) Act, 1972 so Statement 1 is eliminated). 
Eco sensitive zones are area in and around National Park and sanctuaries which are considered as eco fragile zones. There are some activities allowed in these areas like cottage industries, erection of telecom and electricity wires, tourism, hot-air balloons, tourism and so on (so its not just limited to agriculture and Statement 2 is also eliminated).

 

Question : Consider the following pairs

1. Dampa Tiger Reserve : Mizoram
2. Gumti Wildlife Sanctuary : Sikkim
3. Saramati Peak : Nagaland

Which of the above pairs is /are correctly matched?

A. 1 only
B. 2 and 3 only
C. 1 and 3 only
D. 1, 2 and 3

Answer : C

Explanation : Gumti Wildlife Sanctuary is in Tripura 

 

 

 

Question : Wetland locations

Wetlands                                                        Confluence of rivers

 

Harike wetland                                                           Beas and Satluj
Keolado Ghana National Park                                   Banas and Chambal
Kolleru Lake                                                                Musi and Krishna


Which of the above pairs is/are correctly matched? 

A. 1 only
B. 2 and 3 only
C. 1 and 3 only
D. 1, 2 and 3

Answer : A

Explanation : Statement 1 is correct
Statement 2 - Keolado Ghana National Park is Situated at the confluence of the Gambhir and Banganga rivers in the Bharatpur district of Rajasthan
Statement 3 - Kolleru Lake is a  Confluence of Musi and Krishna

 

 

Question : Which of the following have coral reefs?

1. Andaman and Nicobar Islands
2. Gulf of Kutch
3. Gulf of Mannar
4. Sunderbans

Select the correct answer using the code given below.

A. 1, 2 and 3 only
B. 2 and 4 only
C. 1 and 3 only
D. 1, 2, 3 and 4

Answer : A

Explanation : The Coral reefs in India are mainly restricted to the Andaman and Nicobar Islands, Gulf of Mannar, Gulf of Kutch, Palk Strait and the Lakshadweep islands.

Sundarbans have Mangroves and not coral reefs

 

Question : Consider the following:

1. Bats
2. Bears
3. Rodents

The phenomenon of hibernation can be observed in which of the above kinds of animals?

A. 1 and 2 only
B. 2 only
C. 1, 2 and 3
D. Hibernation cannot be observed in any of the above

Answer : C

Explanation : Bats begin hibernating when the cold weather drives the insects away, typically around October and November, and emerge from hibernation in March
Bears hibernate during winter, but not necessarily sleeping the whole time.
Rodents also hibernate 

 

Question : Among the following organisms, which one does not belong to the class of other three?

 

A Crab
B Mite
C Scorpion
D Spider

Answer : A

Explanation : Mite, Scorpion and Spider belong to Arachnida class
Crab belongs to crustacean

 

 

Question : Which one of the following is the correct sequence of a food chain?

A. Diatoms-Crustaceans-Herrings
B. Crustaceans-Diatoms-Herrings
C. Diatoms-Herrings-Crustaceans
D. Crustaceans-Herrings-Diatoms

Answer : A

Explanation : Diatoms are autotrophs which prepare their own food. Crustaceans are herbivorous animals which feed on diatoms. Herrings are carnivorous animals which feed on Crustaceans.

 

Question : If you travel through the Himalayas, you are Likely to see which of the following plants naturally growing there?

1. Oak
2. Rhododendron
3. Sandalwood

Select the correct answer using the code given below

A. 1 and 2 only
B. 3 only
C. 1 and 3 only
D. 1, 2 and 3

Answer : A

Explanation : Oak tree in India are mostly found in the Himalayan region along with pine forest
Rhododendron are also found in the Himalayas
Sandalwood is found in tropical decidous forests or monsoon forests which are found in western ghats, deccan plateu, northern plains and foot hills of Himalayas (not in Himalayas). Hence A is correct

 

Question : If you walk through countryside, you are likely to see some birds stalking alongside the cattle to seize the insects, disturbed by their movement through grasses, Which of the following is/are such bird/birds?

1. Painted Stork
2. Common Myna
3. Black-necked Crane

Select the correct answer using the code given below.

A. 1 and 2
B. 2 only
C. 2 and 3
D. 3 only

Answer : B

Explanation : Painted stork is a fish eating bird and Black Necked Crane feeds on plants and insects in the wild and is it not common of them to feed on insects in the fields. Common myna is an aooportunistic feeder. They feed on insects disturbed by cattle

 

Question : Other than poaching, what are the possible reasons for the decline in the population of Ganges River Dolphins?

1. Construction of dams and barrages on rivers
2. Increase in the population of crocodiles in rivers
3. Getting trapped in fishing nets accidentally
4. Use of synthetic fertilizers and other agricultural chemicals in crop-fields in the vicinity of rivers

Select the correct answer using the code given below.

A. 1 and 2 only
B. 2 and 3 only
C. 1, 3 and 4 only
D. 1, 2, 3 and 4

Answer : C

Explanation : Statements1,3 and 4 are mentioned by WWF . Refer to this article on their website -https://www.worldwildlife.org/species/ganges-river-dolphin

No references found on Statement 2 (crocodiles). Therefore the correct answer is C

 

With reference to two non-conventional energy sources called ‘coal bed methane’ and ‘shale gas’, consider the following ‘statements:

1. Coal bed methane is the pure methane gas extracted from coal seams, while shale gas is a mixture of propane and butane only that can be extracted from fine-grained sedimentary rocks.
2. In India abundant coal bed methane sources exist, but so far no shale gas sources have been found.

Question : Which of the statements given above is/are correct?

A. 1 only
B. 2 only
C. Both 1 and 2
D. Neither 1 nor 2

Answer : D

Explanation : Statement 1 is incorrect - Shale gas consists mainly of methane. Other constituents include hydrocarbons like ethane, propane, and butane, and it also contains carbon dioxide, nitrogen, and hydrogen sulfide.

Statement 2 is incorrect - India has identified six basins as areas for shale gas exploration: Cambay (Gujarat), Assam-Arakan (North East), Gondwana (Central India), Krishna Godavari onshore (East Coast), Cauvery onshore, and Indo-Gangetic basins.

 

 

Question : In India, cluster bean (Guar) is traditionally used as a vegetable or animal feed, but recently the cultivation of this has assumed significance. Which one of the following statements is correct in this context?

A. The oil extracted from seeds is used in the manufacture of biodegradable plastics
B. The gum made from its seeds is used in the extraction of shale gas
C. The leaf extract of this plant has the properties of anti-histamines
D. It is a source of high quality biodiesel

Answer : B

Explanation : It is used in hydraulic fracturing which is used for the extraction of Shale gas. 
Adding guar bean powder (also known as guar gum) to the water increases its viscosity and makes high-pressure pumping and the fracturing process more efficient.

 

 

Question : With reference to technologies for solar power production, consider the following statements:

1. ‘Photovoltaics’ is a technology that generates electricity by direct conversion of light into electricity, while ‘Solar Thermal’ is a technology that utilizes the Sun’s rays to generate heat which is further used in electricity generation process.
2. Photovoltaics generates Alternating Current (AC), while Solar Thermal generates Direct Current (DC).
3. India has manufacturing base for Solar Thermal technology, but not for Photovoltaics.

Which of the statements given above is / are correct?

A. 1 only
B. 2 and 3 only
C. 1, 2 and 3
D. None

Answer : A

Explanation : Photovoltaic technology convert sunlight directly into electricity. Photovoltaics (often shortened as PV) gets its name from the process of converting light (photons) to electricity (voltage), which is called the photovoltaic effect. photovoltaic effect produces a direct current (DC) under solar illumination owing to the directional separation of light‐excited charge carriers at the p–n junction, with holes flowing to the p‐side and electrons flowing to the n‐side

A solar thermal system works by harnessing the sun's energy and converting it into heat which can then be used for electricity. It also generates DC current. 

India has both Solar Thermal technology and Photovoltaic technology manufacturing bases. Recently, Adani Green Energy Ltd that has a 1.5 GW solar Photo Voltaic cell and modules manufacturing capacity bagged a manufacturing-linked solar contract, that entails setting up 2 GW of additional solar cell and module manufacturing capacity.

 

 

Question : There is some concern regarding the nanoparticles of some chemical elements that are used by the industry in the manufacture of various products. Why?

1. They can accumulate in the environment, and contaminate water and soil.
2. They can enter the food chains.
3. They can trigger the production of free radicals.

Select the correct answer using the code given below.

A. 1 and 2 only
B. 3 only
C. 1 and 3 only
D. 1, 2 and 3

Answer : D

Explanation : Statement 1 is possible . Please refer to this writeup for more details - https://www.ncbi.nlm.nih.gov/pmc/articles/PMC5803285/

Statement 1 is correct . Please refer to this writeup for more details - https://link.springer.com/article/10.1007/s42452-019-1412-5?shared-article-renderer

Statement 3 is also correct. Experimental studies show that nanoparticles can trigger the production of free radicals. The chronic release of such reactive molecules can lead to tissue degeneration. Most studies have tested very high concentrations of nanoparticles over relatively short exposure times.
Additional read - https://www.nanowerk.com/nanoparticles_free_radicals_and_oxidative_stress.php#:~:text=Experimental%20studies%20show%20that%20nanoparticles,over%20relatively%20short%20exposure%20times.

 

 

Question : Which of the following are some important pollutants released by steel industry in India?


1. Oxides of sulphur
2. Oxides of nitrogen
3. Carbon monoxide
4. Carbon dioxide

Select the correct answer using the code given below.

A. 1, 3 and 4 only
B. 2 and 3 only
C. 1 and 4 only
D. 1,2, 3 and 4

Answer : D

Explanation : In steel furnace, coke reacts with iron ore, releasing iron and generating CO and CO2 gases. Due to use of coal, pollutants such as SOx and NOx are released, thus all are correct options.

 

 

Question : Brominated flame retardants are used in many household products like mattresses and upholstery. Why is there some concern about their use?

1. They are highly resistant to degradation in the environment.
2. They are able to accumulate in humans and animals.

Select the correct answer using the code given below.

A. 1 only
B. 2 only
C. Both 1 and 2
D. Neither 1 nor 2

Answer : C

Explanation : The widespread production and use of BFRs has strong evidence of increasing contamination of the environment, wildlife, and people; and limited knowledge of potential effects heighten the importance of identifying emerging issues associated with the use of BFRs.

Source - https://www.ncbi.nlm.nih.gov/pmc/articles/PMC1241790/#:~:text=The%20widespread%20production%20and%20use,with%20the%20use%20of%20BFRs.

 

Question : Consider the following countries:

1. Denmark
2. Japan
3. Russian Federation
4. United Kingdom
5. United States of America

Which of the above are the members of the ‘Arctic Council ‘?

A. 1, 2 and 3
B. 2, 3 and 4
C. 1, 4 and 5
D. 1, 3 and 5

Answer : D

Explanation : The Arctic Council consists of the eight Arctic States: Canada, Denmark, Finland, Iceland, Norway, Russia, Sweden and the United States. So, Japan and UK not members.India is also an Observer to the Arctic Council. Hence D is correct

 

Question : With reference to ‘Global Environment Facility’, which of the following statements is/are correct?

A. It serves as financial mechanism for ‘Convention on Biological Diversity’ and ‘United Nations Framework Convention on Climate Change’.
B. It undertakes scientific research on environmental issues at global level
C. It is an agency under OECD to facilitate the transfer of technology and funds to underdeveloped countries with specific aim to protect their environment.
D. Both A and B

Answer : A

Explanation : Option A is correct . Please refer to the GEF webpage - https://www.thegef.org/news/gef-and-convention-biological-diversity
Option B is incorrect (and so Option D too ) - It merely finances the projects but doesn't directly undertake scientific research.
Option C is incorrect as it comes under World Bank.

 

 

Question : If a wetland of international importance is brought under the ‘Montreux Record’, what does it imply?

A. Changes in ecological character have occurred, are occurring or are likely to occur in the wetland as a result of human interference.
B. The country in which the wetland is located should enact a law to prohibit any human activity within five kilo meters from the edge of the wetland
C. The survival of the wetland depends on the cultural practices and traditions of certain communities living in its vicinity and therefore the cultural diversity therein should not be destroyed
D. It is given the status of ‘World Heritage Site’

Answer : A

Explanation : Please refer to https://www.cbd.int/doc/meetings/pa/wscbpa-wanafr-01/other/wscbpa-wanafr-01-presentation-06-en.pdf
In the 11th slide, you would see that Montreux Record: Record of Ramsar Sites “where changes in ecological character have occurred, are occurring or are likely to occur.”

 

 

Question : With reference to a conservation organization called Wetlands International’, which of the following statements is/are correct?

1. It is an intergovernmental organization formed by the countries which are signatories to Ramsar Convention.
2. It works at the field level to develop and mobilize knowledge, and use the practical experience to advocate for better policies.

Select the correct answer using the code given below.

A. 1 only
B. 2 only
C. Both 1 and 2
D. Neither 1 nor 2

Answer : B

Explanation : Statement 1 is incorrect - Wetlands International was formed much before Ramsar Convention although both are in partenership. Wetlands International was formed in 1954 and Ramsar convention was signed in 1971.

Statement 2 is correct - As per their official website (link - https://www.wetlands.org/about-us/ ) "We work at the field level to develop and mobilise knowledge, and use this practical experience to advocate for better policies."

 

 

Question : Consider the following international agreements:

1. The International Treaty on Plant Genetic Resources for Food and Agriculture
2. The United Nations Convention to Combat Desertification
3. The World Heritage Convention

Which of the above has / have a bearing on the biodiversity?

A. 1 and 2 only
B. 3 only,
C. 1 and 3 only
D. 1, 2. and 3

Answer : D

Explanation : The Internation Treaty on Plant Genetic Resources for Food and Agriculture (IT PGRFA), popularly known as the International Seed Treaty is a comprehensive international agreement in harmony with the convention on biological diversity, which aims at guaranteeing security through the conservation.

As per United Nations Convention to Combat Desertification (UNCCD) website, The UNCCD collaborates closely with the other two Rio Conventions - The Convention on Biological Diversity (CBD) as well as the United Nations Framework Convention on Climate Change (UNFCCC)

The World Heritage Convention is one of the most important global conservation instruments which was created in 1972. Its primary mussion is to identify and protect the world's natural and cultural heritage and conservation of biological diversity considered to be of Outstanding Universal Value. 

 

Question : The scientific view is that the increase in global temperature should not exceed 2 °C above pre-industrial level. If the global temperature increases beyond 3°C above the pre-industrial level, what can be its possible impact/impacts on the world?

1. Terrestrial biosphere tends toward a net carbon source
2. Widespread coral mortality will occur.
3. All the global wetlands will permanently disappear.
4. Cultivation of cereals will not be possible anywhere in the world.

Select the correct answer using the code given below.

A. 1 only
B. 1 and 2 only
C. 2, 3 and 4 only
D. 1, 2, 3 and 4,

Answer : B

Explanation : As per IPCC report (IPCC-WG-II, 2007 https://www.ipcc.ch/site/assets/uploads/2018/03/ar4_wg2_full_report.pdf), beyond 30C rise in global temperature above the pre-industrial level will lead to: 

Upto 30% of species at increasing risk of extinction,wide spread coral mortality,

Terrestrial biosphere tends toward a net carbon source Increasing species range shifts and wildfire risk,

Ecosystem changes due to weakening of the meridional overturning circulation. 

So Statements 1 and 2 are correct 

Statement 3 and 4 are too extreme and also IPCC report do not highlight any such risk. Option 4 is wrong because beyond 3 degree rise in global temperature, the cereal crop yield will decrease by 20% in low-altitudes. So saying “cultivation will not be possible” will be wrong as cultivation decreases but doesnt disappear. 

 

Question : Consider the following statements:

1. Animal Welfare Board of India is established under the Environment (Protection) Act, 1986.
2. National Tiger Conservation Authority is a statutory body.
3. National Ganga River Basin Authority is chaired by the Prime Minister.

Which of the statements given above is/ are correct?

A. 1 only
B. 2 and 3 only
C. 2 only
D. 1, 2 and 3

Answer : B

Explanation : Animal Welfare Board was established in 1962 under Section 4 of the Prevention of Cruelty to Animals Act, 1960  So, Statement 1 is wrong and consequently Option A and D (India Yearbook 2014 Pg:29A)

 The National Tiger Conservation Authority (established in 2005) is a statutory body under the Ministry of Environment, Forests and Climate Change.
It was provided statutory status by the Wild Life (Protection) Amendment Act, 2006 which had amended Wild Life (Protection) Act, 1972. Statement 2 is correct.

NGRBA chaired by PM is correct as per India Yearbook 2014 Pg:682. So Statement 3 is correct

So Option B is correct viz. statements 2 and 3

 

 

Question : With reference to Bombay Natural History Society (BNHS), consider the following statements
 
1. It is an autonomous organization under the Ministry of Environment and Forests.
2. It strives to conserve nature through action-based research, education and public awareness.
3. It organizes and conducts nature trails and camps for the general public.

Which of the statements given above is/are correct?

A. 1 and 3 only
B. 2 only
C. 2 and 3 only
D. 1, 2 and 3

Answer : C

Explanation : Statement 1 is incorrect - BNHS is designated as a Scientific and Industrial Research Organization (SIRO) by Department of Science & Technology.(refer to https://www.bnhs.org/who-we-are)

Statement 2 is correct - As per their website (refer to https://www.bnhs.org/who-we-are)
"Our primary goal is to spread awareness about nature through science-based research, conservation advocacy, education, scientific publications, nature tours and other programme"

Statement 3 is correct (refer - https://www.bnhs.org/nature-trails)

Therefore option C is correct. 

 

Question : Consider the following statements regarding ‘Earth Hour’

1. It is an initiative of UNEP and UNESCO.
2. It is a movement in which the participants switch off the lights for one hour on a certain day every year.
3. It is a movement to raise the awareness about the climate change and the need to save the planet.


Which of the statements given above is / are correct?

A. 1 and 3 only
B. 2 only
C. 2 and 3 only.
D. 1, 2 and 3

Answer : C

Explanation : Earth Hour is a worldwide movement organized by the World Wide Fund for Nature (WWF) and so Statement 1 is incorrect .
It is held annually by encouraging individuals, communities, and businesses to turn off non-essential electric lights for one hour on a specific day towards the end of March, as a symbol of commitment to the planet. so Statement 2 is correct 

 

 

Question : Every year, a month long ecologically important campaign/festival is held during which certain communities/ tribes plant saplings of fruit-bearing trees. Which of the following are such communities/tribes?

A. Bhutia and Lepcha
B. Gond and Korku
C. lrula and Toda
D. Sahariya and Agariya

Answer : B

Explanation : The Gond and Korku tribes of Madhya Pradesh's Betul and Harda districts celebrate Hari Jiroti every monsoon . In this month-long festival of greenery, the tribals plant saplings of fruit-bearing trees.
 

APSC Test Series

Attempt an exclusive paper on Environment and Ecology in our APSC GS Prelims Test Series.


Click here to register

UPSC 2015

1. Which one of the following National Parks has a climate that varies from tropical to subtropical, temperate and arctic?
 

(a) Khangchendzonga National park

(b) Nandadevi National Park

(c) Neora Valley National Park

(d) Namdapha National park

Answer (d)

Namdapha National Park is the largest protected area in the Eastern Himalaya biodiversity hotspot and is located in Arunachal Pradesh in Northeast India.

It also harbours extensive dipterocarp forests, comprising the northwestern parts of the Mizoram-Manipur-Kachin rain forests ecoregion.

The habitat changes with increasing altitude from tropical moist forests to Montane forests, temperate forests and at the higher elevations, to Alpine meadows and perennial snow. The park has extensive bamboo forests and secondary forests in addition to the primary forests

2. ‘BioCarbon Fund Initiative for Sustainable Forest Landscapes’ is managed by the
 

(a) Asian Development Bank

(b) International Monetary Fund

(c) United Nations Environment Programme

(d) World Bank

Answer (d)

The BioCarbon Fund Initiative for Sustainable Forest Landscapes (ISFL) is a multilateral facility that promotes and rewards reduced greenhouse gas emissions and increased sequestration through better land management, including REDD+ (Reduced Emissions from Deforestation and forest Degradation), climate smart agriculture, and smarter land use planning and policies. It is managed by World Bank

3. Consider the following statements
 

1. The winds which blow between 30 degrees N and 60 degrees S latitudes throughout the year are

known as westerlies

2. The moist air masses that cause winter rams in North-Western region of India are part of westerlies

Which of the statements given above is/are correct?
 

(a) 1 only

(b) 2 only

(c) Both 1 and 2

(d) Neither 1 nor 2
 

Answer (b)

Westerlies flow between latitudes 30-60 degrees North and 30-60 degrees South. The statement mentions 30N-60S. So it is wrong.. They originate from the high-pressure areas in the horse latitudes and tend towards the poles and steer extra tropical cyclones in this general manner. The moist air masses that cause winter rains in North-Western region of India are part of Westerlies.

4. With reference to ‘Forest Carbon Partnership Facility’, which of the following statements is/are correct?
 

1. It is global partnership of governments, businesses, civil society and indigenous peoples

2. It provides financial aid to universities, individual scientists and institutions involved in scientific

forestry research to develop eco-friendly and climate adaptation technologies for sustainable forest

management

3. It assists the countries in their ‘REDD+ (Reducing Emission from Deforestation and Forest

Degradation)’ efforts by providing them with financial and technical assistance

Select the correct answer using the code given below
 

(a) 1 only

(b) 2 and 3 only

(c) 1 and 3 only

(d) 1, 2 and 3

Answer (c)

The Forest Carbon Partnership Facility is a global partnership of governments, businesses, civil society, and Indigenous Peoples focused on reducing emissions from deforestation and forest degradation, forest carbon stock conservation, the sustainable management of forests, and the enhancement of forest carbon stocks in developing countries (activities commonly referred to as REDD+).

5. With reference to an organization known as ‘Birdlife International’ which of the following statements is/are correct?

1. It is a Global Partnership of Conservation Organizations.
2. The concept of ‘biodiversity hotspots’ originated from this organization.
3. It identifies the sites known/referred to as ‘Important Bird and Biodiversity Areas’.
Select the correct answer using the code given below.

a) 1 only
b) 2 and 3 only
c) 1 and 3 only
d) 1, 2 and 3
 

Answer (c)

BirdLife International is a global partnership of conservation organizations (NGOs) that strives to conserve birds, their habitats, and global biodiversity, working with people towards sustainability in the use of natural resources. Hence statement 1 is correct.

The concept of biodiversity hotspots was developed by Norman Myers in 1988. Hence statement 2 is incorrect.

The IBA Programme of BirdLife International aims to identify, monitor, and protect a global network of IBAs for the conservation of the world's birds and another biodiversity. Hence statement 3 is correct.

 

As of 2009, nearly 11,000 sites in some 200 countries and territories have been identified as Important Bird Areas. BirdLife Partnership works collectively to identify, document, and protect the places of the greatest significance for the conservation of Important Bird and Biodiversity Areas (IBAs).

6. Which one of the following is the best description of the term “ecosystem”?
 

a) A community of organisms interacting with one another
b) That part of the Earth which is inhabited by living organisms
c) A community of organisms together with the environment in which they live.
d) The flora and fauna of a geographical area.
 

Answer (c)

An ecosystem is a community of living organisms in conjunction with the non-living components of their environment, interacting as a system. These biotic and abiotic components are regarded as linked together through nutrient cycles and energy flows.

7. Which of the following National Parks is unique in being a swamp with floating vegetation that supports a rich biodiversity?
 

(a) Bhitarkanika National Park

(b) Keibul Lamjao National Park

(c) Keoladeo Ghana National park

(d) Sultanpur National park

 

Answer (b)

It is the only floating park in the world, located in Manipur, North East India, and an integral part of Loktak Lake.

The national park is characterized by many floating decomposed plant materials locally called phumdis.

The park is a swamp established by Mann Sharma with floating mass of vegetation (created by accrual of organic garbage and biomass with soil particles that has been thickened into a solid form called phumdis, at the south–eastern side of the Loktak Lake, which has been declared a Ramsar site. Two third’s to three fourth’s of the total park area is formed by phumdis.

8. With reference to the International Union for Conservation of Nature and Natural Resources (IUCN) and the Convention on International Trade in Endangered Species of Wild Fauna and Flora (CITES), which of the following statements is/are correct?
 

1. IUCN is an organ of the United Nations and CITES is an international agreement between governments

2. IUCN runs thousands of field projects around the world to better manage natural environments

3. CITES is legally binding on the States that have joined it, but this Convention does not take the place of national laws
 

Select the correct answer using the code given below.

(a) 1 only

(b) 2 and 3 only

(c) 1 and 3 only

(d) 1, 2 and 3

 

Answer (b)

 

IUCN is a voluntary organization, not an agency of United Nations. Even if you knew this much, you could eliminate all other options to select B. Some facts about IUCN:

• Founded in 1948 as the world’s first global environmental organisation

• Today the largest professional global conservation network

• A leading authority on the environment and sustainable development

• More than 1,200 member organizations including 200+ government and 900+ non-government organizations

• A neutral forum for governments, NGOs, scientists, business and local communities to find practical solutions to conservation and development challenges

• Thousands of field projects and activities around the world

CITES was drafted as a result of a resolution adopted in 1963 at a meeting of members of the International Union for Conservation of Nature (IUCN). States (countries) adhere voluntarily to the agreement.

States that have agreed to be bound by the Convention (‘joined’ CITES) are known as Parties.

Although CITES is legally binding on the Parties – in other words they have to implement the Convention – it does not take the place of national laws. Rather it provides a framework to be respected by each Party, which has to adopt its own domestic legislation to ensure that CITES is implemented at the national level.

9. With reference to ‘dugong’, a mammal found in India, which of the following statements is/are correct?
 

1. It is a herbivorous marine animal

2. It is found along the entire coast of India

3. It is given legal protection under Schedule 1 of the Wildlife (Protection) Act, 1972
 

Select the correct answer using the code given below.
 

a) 1 and 2

b) 2 only

c) 1 and 3

d) 3 only

 

Answer (c)

Dugong, Dugong dugon also called ‘Sea Cow’ is one of the four surviving species in the Order Sirenia and it is the only existing species of herbivorous mammal that lives exclusively in the sea including in India. 
 

Dugongs are protected in India and occur in the Gulf of Mannar, PalkBay, Gulf of Kutch and Andaman and Nicobar islands.
 

Once abundant in Indian waters, the Dugong population has now reduced to about 200 individuals and is believed to be continuously declining in its number and range.
 

Dugong conservation is nothing but coastal conservation being a flagship species in its range. In order to conserve and manage the declining populations of dugong in India, the Ministry of Environment, Forests and Climate Change under the Government of India constituted a ‘Task Force for Conservation of Dugongs’ to look into the entire gamut of issues related to the conservation of dugongs and implementation of the ‘UNEP/CMS Dugong MoU’ in India and also to facilitate the country to act as the leading nation in the South Asia Sub-region with respect to dugong conservation.

The Task Force has analyzed the entire spectrum of issues linked to dugong and their habitat conservation in India and herewith recommends the following goals and objectives for Conservation and Management Plan for Dugongs (Dugong dugon) and their habitats in India.
 

It is given legal protection under Schedule I of the Wildlife (Protection) Act, 1972. Vulnerable in IUCN Red List

10. Which one of the following is the national aquatic animal of India?
 

(a)Saltwater crocodile

(b) Olive ridley turtle

(c) Gangetic dolphin

(d) Gharial

 

Answer (c)

River Dolphin is the National Aquatic Animal of India. This mammal is also said to represent the purity of the holy Ganga as it can only survive in pure and fresh water.
They are locally known as susu, because of the noise it makes while breathing. This species inhabits parts of the Ganges, Meghna and Brahmaputra rivers in India, Nepal, Bhutan and Bangladesh, and the Karnaphuli River in Bangladesh.
River dolphin is a critically endangered species in India and therefore, has been included in the Schedule I for the Wildlife (Protection) Act, 1972. The main reasons for decline in population of the species are poaching and habitat degradation due to declining flow, heavy siltation, construction of barrages causing physical barrier for this migratory species.

11. What is Rio+20 Conference, often mentioned in the news?
 

(a) It is the United nations Conference on Sustainable Development

(b) It is a Ministerial Meeting of the World Trade Organization

(c)It is a Conference of the Inter-governmental Panel on Climate Change

(d) It is a Conference of the Member Countries of the Convention on Biological Diversity


Answer (a)

The United Nations Conference on Sustainable Development (UNCSD), also known as Rio 2012, Rio+20, or Earth Summit 2012 was the third international conference on sustainable development aimed at reconciling the economic and environmental goals of the global community. Hosted by Brazil in Rio de Janeiro from 13 to 22 June 2012, Rio+20 was a 20-year follow-up to the 1992 Earth Summit / United Nations Conference on Environment and Development (UNCED).

12. The term ‘Goldilocks Zone’ is often seen in the news in the context of
 

(a) the limits of habitable zone above the surface of the Earth

(b) regions inside the Earth-like planets in outer space

(c) search for the Earth-like planets in outer space

(d) search for meteorites containing precious metals

Answer (c)

In astronomy and astrobiology, the circumstellar habitable zone (CHZ), or simply the habitable zone (Goldilocks Zone), is the region around a star within which planetary-mass objects with sufficient atmospheric pressure can support liquid water at their surfaces.

Researchers are finding that life can thrive in some unexpected places.

12. Which of the following statements regarding ‘Green Climate Fund’ is/are correct?
 

1. It is intended to assist the developing countries in adaptation and mitigation practices to counter

climate change

2. It is founded under the aegis of UNEP, OECD, Asian Development Bank and World Bank

Select the correct answer using the code given below.
 

(a) 1 only

(b) 2 only

(c) Both 1 and 2

(d) Neither 1 nor 2

 

Answer (a)

The Green Climate Fund (GCF) is a fund within the framework of the UNFCCC founded as a mechanism to redistribute money from the developed to the developing world, in order to assist the developing countries in adaptation and mitigation practices to counter climate change.

The Green Climate Fund was designated as an operating entity of the financial mechanism of the UNFCCC, in accordance with Article 11 of the Convention. Arrangements will be concluded between the Conference of the Parties (COP) and the Fund to ensure that it is accountable to, and functions under the guidance of, the COP.

The Fund is governed and supervised by a Board that will have full responsibility for funding decisions and that receives the guidance of the COP.

UPSC 2016

1. Which of the following best describes/ describe the aim of the ‘Green India Mission’ of the Government of India?
 

1. Incorporating environmental benefits and costs into the Union and State Budgets thereby implementing the `green accounting’

2. Launching the second green revolution to enhance agricultural output so as to ensure food security to one and all in the future

3. Restoring and enhancing forest cover and responding to climate change by a combination of adaptation and mitigation measures
 

Select the correct answer using the code given below.
 

(a) 1 only
(b) 2 and 3 only

(c) 3 only

(d) 1, 2 and 3

 

Answer (c)

 

The National Mission for a Green India was announced by the Prime Minister as one of the eight Missions under the National Action Plan on Climate Change (NAPCC). It recognizes that climate change phenomenon will seriously affect and alter the distribution, type and quality of natural resources of the country. GIM puts the “greening” in the context of climate change adaptation and mitigation, meant to enhance ecosystem services like carbon sequestration and storage (in forests and other ecosystems), hydrological services and biodiversity; along with provisioning services like fuel, fodder, small timber.

 

Ref: India Yearbook 2016 page 338: Environment ministry has launched National Mission for a Green India through a consultative process involving relevant stakeholders, aimed at both increasing the forest and tree cover by 5 million ha, as well as increasing the quality of the existing forest cover in another 5 million ha. Hence only statement 3 fits

2. What is/are unique about ‘Kharai Camel’, a breed found in India?
 

1. It is capable of swimming up to three kilometres in seawater

2. It survives by grazing on mangroves

3. It lives in the wild and cannot be domesticated
 

Select the correct answer using the code given below.
 

(a) 1 and 2 only

(b) 3 only

(c) 1 and 3 only

(d) 1, 2 and 3

 

Answer (a)

 

 

Statement 1 and 2: It can swim up to three kilometers into the sea in search of mangroves, its primary food.

 

The breed population has declined drastically in recent years due to loss of mangroves.

 

Statement 3: Given the breed’s ability to survive both on land and sea, the Kharai camel is one of the most preferred choices of graziers in the arid coastal region of Kachchh.

 

People consume its milk, while male calves are sold for economic returns (females are not sold because they are considered sacred). So, clearly they are domesticated and don’t live in the wild. Statement 3 is wrong.

 

Gujarat is the only home to Kharai camel.

3. Recently, our scientists have discovered a new and distinct species of banana plant which attains a

height of about 11 metres and has orange-coloured fruit pulp. In which part of India has it been discovered?
 

(a) Andaman Islands

(b) Anaimalai Forests

(c) Maikal Hills

(d) Tropical rain forests of northeast
 

Answer (a)

Justification: Musa indandamanensis, a sweet species of banana with orange pulp, has been discovered in the islands of Andamans.

The banana species was found by a team of scientists from the Botanical Survey of India (BSI) from the Krishna Nalah tropical rain forest on the islands.

It has also been reported by the scientists that the discovered species is thrice the size of a regular banana species

4. With reference to an initiative called ‘The Economics of Ecosystems and Biodiversity (TEEB)’, which of the following statements is/are correct?
 

1. It is an initiative hosted by UNEP, IMF and World Economic Forum

2. It is a global initiative that focuses on drawing attention to the economic benefits of biodiversity

3. It presents an approach that can help decision-makers recognize, demonstrate and capture the value of ecosystems and biodiversity
 

Select the correct answer using the code given below.
 

(a) 1 and 2 only

(b) 3 only

(c) 2 and 3 only

(d) 1, 2 and 3

 

Answer (c)

 

The Economics of Ecosystems and Biodiversity (TEEB) is a study led by Pavan Sukhdev. It is an international initiative to draw attention to the global economic benefits of biodiversity. Its objective is to highlight the growing cost of biodiversity loss and ecosystem and to draw together expertise from the fields of science, economics and policy to enable practical actions.

As per the official page of the organization, 2 and 3 are correct, but there is no mention of their association with UNEP, IMF. Hence 1st statement is wrong. Accordingly answer is “c”

5. With reference to ‘Red Sanders’, sometimes seen in the news, consider the following statements:
 

1. It is a tree species found in a part of South India

2. It is one of the most important trees in the tropical rainforest areas of South India

Which of the statements given above is/are correct?
 

(a) 1 only
(b) 2 only

(c) Both 1 and 2

(d) Neither 1 nor 2

 

Answer (a)

Statement 1: Red Sanders (a tree) has a highly restrictive distribution in the South Eastern portion of Indian peninsula (Eastern Ghats of South India) to which it is endemic. So, clearly 1 is correct.

Statement 2: It occurs in the forest formation which is classified as “Southern Tropical Dry Deciduous Forests”. So, clearly 2 is wrong.

It is generally found at altitudes of 150 – 900 m. It grows on dry, hilly, often rocky ground, and occasionally found on precipitous hill sides also. It prefers lateritic and gravelly soil and cannot tolerate water logging.

6. Which of the following statements is/are correct?
 

Proper design and effective implementation of UN-REDD+ Programme can significantly contribute to
 

1. protection of biodiversity

2. resilience of forest ecosystems

3. poverty reduction
 

Select the correct answer using the code given below.
 

(a) 1 and 2 only

(b) 3 only

(c) 2 and 3 only

(d) 1, 2 and 3

 

Answer (d)

 

The REDD+, Reducing emissions from deforestation and forest degradation and the role of conservation, sustainable management of forests and enhancement of forest carbon stocks in developing countries (REDD+) was first negotiated under the United Nations Framework Convention on Climate Change (UNFCCC) in 2005.

The main objective was to mitigate climate change through reducing net emissions of greenhouse gases through enhanced forest management in developing countries.

Most of the key REDD+ decisions were completed by 2013, with the final pieces of the rulebook finished in 2015.

It is based off the assumption that without REDD+, the forests in the developing countries have been cut down, accelerating climate change.

Proper implementation of the REDD+ strategies can help in tackling:

  • Forest degradation and biodiversity loss.

  • Improving Social standards reducing poverty.

  • Resilience of forest ecosystem.

7. With reference to ‘Agenda 21’, sometimes seen in the news, consider the following statements :
 

1. It is a global action plan for sustainable development

2. It originated in the World Summit on Sustainable Development held in Johannesburg in 2002
 

Which of the statements given above is/are correct?
 

(a) 1 only

(b) 2 only

(c) Both 1 and 2

(d) Neither 1 nor 2

 

Answer (a)

Agenda 21 is a non-binding, voluntarily implemented action plan of the United Nations with regard to sustainable development.

It is a product of the Earth Summit (UN Conference on Environment and Development) held in Rio de Janeiro, Brazil, in 1992.

8. Which of the following statements is/are correct?

Viruses can infect
 

1. bacteria

2. fungi

3. plants
 

Select the correct answer using the code given below.
 

(a) 1 and 2 only

(b) 3 only

(c) 1 and 3 only

(d) 1, 2 and 3

 

Answer (d)

Viruses are known to infect almost any kind of host that has living cells. Animals, plants, fungi, and bacteria are all subject to viral infection.

A bacteriophage is a virus that infects and replicates within a bacterium. Mycoviruses are viruses that infect fungi. A host of other Viruses affect plants.

9. In the cities of our country, which among the following atmospheric gases are normally considered in calculating the value of Air Quality Index?

1. Carbon dioxide

2. Carbon monoxide

3. Nitrogen dioxide

4. Sulfur dioxide

5. Methane
 

Select the correct answer using the code given below.
 

(a) 1, 2 and 3 only

(b) 2, 3 and 4 only

(c) 1, 4 and 5 only

(d) 1, 2, 3, 4 and 5

 

Answer (b)

The API level is based on the level of 6 atmospheric pollutants, namely sulfur dioxide (SO2), nitrogen dioxide (NO2), suspended particulates smaller than 10 μm in aerodynamic diameter (PM10), suspended particulates smaller than 2.5 μm in aerodynamic diameter (PM2.5), carbon monoxide (CO), and ozone (O3) measured at the monitoring stations throughout. So by elimination we are left with option “b”.

10. Consider the following statements:
 

1. The Sustainable Development Goals were first proposed in 1972 by a global think tank called the ‘Club of Rome’

2. The Sustainable Development Goals have to be achieved by 2030

Which of the statements given above is/are correct?
 

(a) 1 only

(b) 2 only

(c) Both 1 and 2

(d) Neither 1 nor 2

 

Answer (b)

The first report of the Club of Rome was the famous The limits to growth, brought out in 1972 by an MIT research team coordinated by Dennis and Donella Meadows.

The Sustainable Development Goals (SDGs) were adopted by all United Nations Member States in 2015 to end poverty, reduce inequality and build more peaceful, prosperous societies by 2030. Hence statement 2 is correct.

The objective of SDG was to produce a set of universal goals that meet the urgent environmental, political, and economic challenges facing our world.

Sustainable Development Goals (SDGs) were born at the United Nations Conference on Sustainable Development in Rio de Janeiro in 2012. Hence statement 1 is incorrect.

11. In which of the following regions of India are you most likely to come across the `Great Indian Hornbill’ in its natural habitat?
 

(a) Sand deserts of northwest India

(b) Higher Himalayas of Jammu and Kashmir

(c) Salt marshes of western Gujarat

(d) Western Ghats

 

Answer (d)

Great Hornbills are found in three separate areas in South Asia; in the Western Ghats, the Himalayan foothills in Uttaranchal to south Nepal and Bhutan, and north-east India.

They occur in primary evergreen and moist deciduous forest, mainly in lowland forest, but they can be found up to 2,000 meters in some areas.

All other options apart from D can thus be easily eliminated.

12. Recently, for the first time in our country, which of the following States has declared a particular butterfly as ‘State Butterfly’?
 

(a) Arunachal Pradesh

(b) Himachal Pradesh

(c) Karnataka

(d) Maharashtra

 

Answer (d)
Factual - http://www.thehindu.com/news/national/other-states/maharashtra-gets-state-butterfly/article7342955.ece

UPSC 2017

1. Consider the following statements in respect of Trade Related Analysis of Fauna and Flora in Commerce (TRAFFIC):
 

1. TRAFFIC is a bureau under United Nations Environment Programme (UNEP)

2. The mission of TRAFFIC is to ensure that trade in wild animals is not a threat to the conservation of nature
 

Which of the given statements is/are correct?
 

(a) 1 only

(b) 2 only

(c) Both 1 and 2

(d) Neither 1 nor 2

 

Answer (b)

Statement 1: It is a NGO, not a bureau under UNEP. So, 1 is wrong.

TRAFFIC is governed by the TRAFFIC Committee, a steering group composed of members of TRAFFIC’s partner organizations, WWF and IUCN.

Statement 2: It is working globally on Wildlife trade monitoring network.

It specializes in investigating and analysing wildlife trade trends, patterns, impacts and drivers to provide the leading knowledge base on trade in wild animals and plants.

2. In the context of solving pollution problems, which is/are the advantage/advantages of bioremediation techniques?
 

1. It is a technique of cleaning up pollution by enhancing the same biodegradation process that occurs in nature

2. Any contaminant with heavy metals such as cadmium and lead can be readily and completely

treated by bioremediation using microorganisms

3. Genetic engineering can be used to create microorganisms specifically designed for bioremediation

Select the correct answer using the codes given below:

(a) 1 only

(b) 2 and 3 only

(c) 1 and 3 only

(d) 1, 2, and 3

Answer (c)

 

Bioremediation technique -
 

Bioremediation is a treatment that uses naturally occurring organisms to break down hazardous substances into less toxic or non-toxic substances. It uses microorganisms to degrade organic contaminants in soil, groundwater, sludge, and solids.

  - The bioremediation technique is a technique for cleaning up pollution by enhancing the same biodegradation process that occurs in nature. So statement 1 is correct.
 

Not all contaminants are easily treated by bioremediation using microorganisms. For example, heavy metals such as cadmium and lead are not readily absorbed or captured by microorganisms.

  -Bioremediation is limited to biodegradable compounds. So statement 2 is incorrect.

  -Bioremediation is the use of microorganisms mainly bacteria and fungi to degrade the environmental contaminants into less toxic forms.

Genetic engineering can be used to create microorganisms specifically designed for bioremediation. So statement 3 is correct.

3. Due to some reasons if there is a huge fall in the population of a species of butterflies, what could be its likely consequence/consequences?

 

1. Pollination of some plants could be adversely affected

2. There could be a drastic increase in the fungal infection of some cultivated plants

3. It could lead to a fall in the population of some species of wasps, spiders and birds

 

Select the correct answer using the code given below:

 

(a) 1 only

(b) 2 and 3 only

(c) 1 and 3 only

(d) 1, 2 and 3
 

Answer (c)

Statement 1: Bees and butterflies play a vital role in the pollination of plants and the production of crops by transporting pollen grains from one place to another.

Statement 2: This is not a valid statement.

Statement 3: These are some of the common predators of butterflies: wasps, ants, parasitic flies, birds, snakes, toads, rats etc. The decline in butterfly population would therefore adversely affect the food chain.

4. Is it possible to produce algae based biofuels, but what is/are the likely limitation(s) of developing countries in promoting this industry?

1. Production of algae based biofuels is possible in seas only and not on continents

2. Setting up and engineering the algae based biofuel production requires high level of expertise/

technology until the construction is completed

3. Economically viable production necessitates the setting up of a large scale facilities which may raise ecological and social concerns
 

Select the correct answer using the codes given below:
 

(a) 1 and 2 only

(b) 2 and 3 only

(c) 3 only

(d) 1, 2 and 3

Answer (b)

Statement 1: Algaculture (farming algae) can be initiated on land unsuitable for agriculture or saline water or wastewater. So, 1 is wrong.

Statement 2: Producing algae for bio-fuels (excluding simple seaweed production) requires significant capital investments, which may be a significant barrier in developing countries that have a weak investment climate.

Producing biofuels from algae is still in a pre-commercial state of technology development. Most production plants are therefore prototypes, and the development and engineering of such plants requires a high level of expertise.

Statement 3: Using the land allotted to food crops to algal biofuel reduces the amount of food available for humans, resulting in an increased cost for both the food and the fuel produced. This is an ecological as well as economic concern.

5. In the context of mitigating the impending global warming due to anthropogenic emissions of carbon dioxide, which of the following can be the potential sites for carbon sequestration?
 

1. Abandoned and uneconomic coal seams

2. Depleted oil and gas reservoirs

3. Subterranean deep saline formalities
 

Select the correct answer using the codes given below:
 

(a) 1 and 2 only

(b) 3 only
(c) 1 and 3 only

(d) 1, 2 and 3

 

Answer (d)

 

What is Carbon sequestration?

The process of storing (or capturing) atmospheric carbon dioxide is called Carbon sequestration. It is used for reducing the amount of carbon dioxide in the atmosphere.

 

Its uses/methods:

Abandoned and uneconomic coal seams can be used to store CO2 as the CO2 molecules attach to the surface of coal. So statement 1 is correct.

CO2 is also used as the recovery tool as by injecting CO2 into declining oil fields to increase oil recovery. So statement 2 is correct.

Forests as sink as trees are natural sequesters of carbon. Wetland restoration as wetlands conserves 14.5 % of the soil carbon found in the world.

Also used for subterranean deep saline formations. So statement 3 is correct.

6. Consider the following statements:
 

1. In tropical regions, Zika virus disease is transmitted by the same mosquito that transmits dengue

2. Sexual transmission of Zika virus disease is possible
 

Which of the given statements is/are correct?

(a) 1 only

(b) 2 only

(c) Both 1 and 2

(d) Neither 1 nor 2

 

Answer (c)

Statement 1:  Zika virus is transmitted to people primarily through the bite of an infected Aedes species mosquito (Ae. aegypti and Ae. albopictus). These are the same mosquitoes that spread dengue and chikungunya viruses.

Statement 2:  Zika can be passed through sex from a person who has Zika to his or her partners. Zika can be passed through sex, even if the infected person does not have symptoms at the time.

7. According to the Wildlife (Protection) Act, 1972, which of the following animals cannot be hunted by any person except under some provisions provided by law?
 

1. Gharial

2. Indian wild ass

3. Wild Buffalo
 

Select the correct answer using the code given below:
 

(a) 1 only

(b) 2 and 3 only

(c) 1 and 3 only

(d) 1, 2 and 3

Answer (d)

All of these are protected scheduled animals under the Wildlife Act.

8. With reference to ‘Global Climate Change Alliance’, which of the following statements is/are correct?
 

1. It is an initiative of the European Union

2. It provides technical and financial support to targeted developing countries to integrate climate change into their development budgets

3. It is coordinated by World Resources Institute (WRI) and World Business Council for Sustainable Development (WBCSD)
 

Select the correct answer using the code given below:
 

(a) 1 and 2 only

(b) 3 only

(c) 2 and 3 only

(d) 1, 2 and 3

 

Answer (a)

Statement 1: The GCCA was established by the European Union (EU) in 2007 to strengthen dialogue and cooperation with developing countries, in particular least developed countries (LDCs) and small island developing States (SIDS).

Statement 2: By fostering effective dialogue and cooperation on climate change, the Alliance helps to ensure that poor developing countries most vulnerable to climate change increase their capacities to adapt to the effects of climate change, in support of the achievement of the Millenium Development Goals (MDGs).

Statement 3: There is no mention of WRI and WBCSD at the official partner’s page of GCCA, even though a lot of other institutions are mentioned like FAO, UNDP etc.

9. With reference to the role of UN-Habitat in the United Nations programme working towards a better urban future, which of the following statements is/are correct?
 

1. UN-Habitat has been mandated by the United Nations General Assembly to promote socially and environmentally sustainable towns and cities to provide adequate shelter for all

2. Its partners are either governments or local urban authorities only

3. UN-Habitat contributes to the overall objective of the United Nations system to reduce poverty and to promote access to safe drinking water and basic sanitation
 

Select the correct answer from the code given below:
 

(a) 1, 2 and 3

(b) 1 and 3 only

(c) 2 and 3 only

(d) 1 only

 

Answer (b)

UN-Habitat has been mandated by the United Nations General Assembly.

UN-Habitat has served as the global leader in promoting socially and environmentally sustainable human settlements and adequate shelter for all. (Hence statement 1 is correct)

UN-Habitat collaborates with governments, intergovernmental, UN agencies, civil society organizations, foundations, academic institutions and including the private sector to achieve enduring results in addressing the challenges of urbanization. (Hence statement 2 is not correct)

Un-Habitat was established in 1978.

UN-Habitat contributes to the overall objective of the United Nations system to reduce poverty and to promote access to safe drinking water and basic sanitation. (Hence statement 3 is correct).

10. Biological Oxygen Demand (BOD) is a standard criteria for
 

(a) Measuring oxygen level in blood

(b) Computing oxygen levels in forest ecosystems

(c) Pollution assay in aquatic ecosystem

(d) Assessing oxygen levels in high altitude regions

 

Answer (c)

It is the amount of dissolved oxygen needed (i.e., demanded) by aerobic biological organisms to break down organic material present in a given water sample at certain temperature over a specific time period.

11. Consider the following statements:
 

1. Climate and Clean Air Coalition (CCAC) to reduce Short Lived Climate Pollutants is a unique initiative of G20 group of countries

2. The CCAC focusses on methane, black carbon and hydrochlorofluorocarbons
 

Which of the given statements given above is/are correct?
 

(a) 1 only

(b) 2 only

(c) Both 1 and 2

(d) Neither 1 nor 2

 

Answer (b)

Statement 1:  The Climate and Clean Air Coalition to Reduce Short-Lived Climate Pollutants (CCAC) was launched by the United Nations Environment Programme (UNEP) and six countries—Bangladesh, Canada, Ghana, Mexico, Sweden, and the United States—in 2012.

Statement 2:  The Coalition’s initial focus is on methane, black carbon, and HFCs.

12. If you want to see gharials in their natural best habitat, which one of the following is the best place to visit?
 

(a) Bhitarkanika Mangroves

(b) Chambal River

(c) Pulicat Lake

(d) Deepor Beel

 

Answer (b)

A sanctuary for gharials (freshwater crocodiles) spread over 1600 sq km created along the Chambal River.

- More than 5,000 gharials (Gavialis gangeticus) were born in the latest hatching season (June-July) at the National Chambal Sanctuary on the tri-junction of Uttar Pradesh, Madhya Pradesh and Rajasthan.

- Apart from gharials, 400 mugger crocodiles (Crocodylus palustris) also hatched out of eggs at the sanctuary and will soon be released into the river.

- Scientist believe that Red-crowned turtle and birds like the Indian skimmer on this stretch of the Chambal are more threatened than gharials.

13. The term M-STrIPES’ is sometimes seen in news in the context of
 

(a) Captive breeding of Wild Fauna

(b) Maintenance of Tiger Reservoirs

(c) Indegenous Satellite Navigation System

(d) Security of National Highways
 

Answer (b)

The android-based monitoring software M-STrIPES will be used across all the Tiger Reserves of the country.

14. In India, if a species of tortoise is declared protected under Schedule I of the Wildlife (Protection) Act, 1972, what does it imply?
 

(a) It enjoys the same level of protection as tiger

(b) It no longer exists in the wild, a few individuals are under captive protection; and now it is impossible to prevent its extinction

(c) It is endemic to a particular region of India

(d) Both (b) and (c) stated above are correct in this context

 

Answer (a)

Schedule I animals under the Wildlife Act enjoy the highest protection in India. Since tiger is also covered in Schedule I, option A is correct.

15. Recently, there was a proposal to translocate some of the lions from their natural habitat in Gujarat to which one of the following sites?

 

(a) Corbett National Park

(b) Kuno Palpur Wildlife Sanctuary

(c) Mudumalai Wildlife Sanctuary

(d) Sariska National Park

 

Answer (b)

In news and mired by controversies due to Gujarat government’s reluctance to send tigers to MP.

The Supreme Court in 2013 ruled in favour of translocation of the Asiatic lion to Kuno in the interest of the genetic stability of the species.

http://economictimes.indiatimes.com/news/politics-and-nation/gujarat-swallows-its-pride-may-agree-to-translocate-gir-lions-to-kuno-sanctuary/articleshow/52803382.cms

UPSC 2018

1. Which one of the following is an artificial lake?
 

(a) Kodaikanal (Tamil Nadu)

(b) Kolleru (Andhra Pradesh)

(c) Nainital (Uttarakhand)

(d) Renuka (Himachal Pradesh)

 

Answer (a)

Kodaikanal Lake, also known as Kodai Lake is a manmade lake located in the Kodaikanal city in Dindigul district in Tamil Nadu, India. Kodaikanal Lake is in the Palni Hills.

It is starfish-shaped but with four points, centrally located in the town of Kodaikanal and is surrounded by lush green hills of the Northwestern Palani Hills.

2. Which of the following is/are the possible consequences of heavy and mining in riverbeds?
 

1. Decreased salinity in the river

2. Pollution of groundwater

3. Lowering of the water table
 

Select the correct answer using the code given below:
 

(a) 1 only

(b) 2 and 3 only

(c) 1 and 3 only

(d) 1, 2 and 3

 

Answer (b)

The removal of sand from the river bed increases the velocity of the flowing water, with the distorted flow-regime eventually eroding the river banks.

Sand acts like a sponge, which helps in recharging the water table; its progressive depletion in the river is accompanied by declining water tables in the nearby areas.

Depletion of sand in the stream bed causes the deepening of rivers and estuaries, and the enlargement of river mouths and coastal inlets. It leads to saline-water intrusion.

Sand acts as an efficient filter for various pollutants and thus maintains the quality of water in rivers and other aquatic ecosystems.

3. The Partnership for Action on Green Economy (PAGE) a UN mechanism to assist countries transition towards greener and more inclusive economies, emerged at
 

(a) The Earth Summit on Sustainable Development 2002, Johannesburg

(b) The United Nations Conference on Sustainable Development 2012, Rio de Janeiro

(c) The United Nations Framework Convention on Climate Change 2015, Paris

(d) The World Sustainable Development Summit 2016, New Delhi

 

Answer (b)

In 2012, Rio+20 (the United Nations Conference on Sustainable Development) was held in Brazil. The conference’s outcome document entitled ‘The Future We Want’ was a call to action for governments, business and the UN alike to support countries interested in transition to a green economy. PAGE was created as the UN’s direct response to this call of action.

Four UN organizations, including ILO, UN Environment, UNIDO and UNITAR initially joined forces to be able to provide countries with broad and tailored support.

4. Why is a plant called Prosopis Juliflora often mentioned in the news?
 

(a) Its extract is widely used in cosmetics

(b) It tends to reduce the biodiversity in the area in which it grow

(c) Its extract is used in the synthesis of pesticides

(d) None of the above

 

Answer (b)

The kikar or Prosopis juliflora was brought to Delhi from Mexico by the British more than a century ago. The exotic plant became invasive and wiped out most of the native plants and along with it the animals, which once used to roam in the ridges. With its deep roots, it had also wreaked havoc on city’s groundwater.

5. Consider the following statements:
 

1. The Earth’s magnetic field has reversed every few hundred thousand years

2. When the Earth was created more than 4000 million years ago, there was 54% oxygen and no carbon dioxide

3. When living organisms originated, they modified the early atmosphere of the Earth
 

Which of the statements given above is/are correct?
 

(a) 1 only

(b) 2 and 3 only

(c) 1 and 3 only

(d) 1, 2 and 3

 

  • Scientists estimate reversals have happened at least hundreds of times over the past three billion years.

  • Over the last 20 million years, magnetic north and south have flipped roughly every 200,000 to 300,000 years.

  • The last of these major reversals occurred about 780,000 years ago.

  • Between 4.5 and 2.5 billion years (the Archaean and Proterozoic time), the earliest secondary atmosphere contained carbon dioxide (CO2), methane (CH4), water vapor (H2O), carbon monoxide (CO), a little nitrogen (N), and hydrogen (H). So Carbon dioxide is present and hence statement 2 is wrong.

  •  When living organisms originated, they did not affect the early atmosphere. However, according to the official answer key, the answer is Option 3, i.e 1 and 3 only.

6. In which one of the following states is Pakhui wildlife sanctuary located?
 

(a) Arunachal Pradesh

(b) Manipur

(c) Meghalaya

(d) Nagaland
 

Answer (a)

Spread over an area of 862 sq. km. at the foothills of the Eastern Himalayas in Arunachal Pradesh, Pakhui Wildlife Sanctuary is also known as Pakke Wildlife Sanctuary and more popularly as Pakke Tiger Reserve. Earlier a game sanctuary, it became a wildlife sanctuary in 2001 and a part of Project Tiger in 2002. The sanctuary is well-known for being home to tigers and many amazing species of hornbill.

7. Consider the following statements?
 

1. The definition of “Critical Wildlife Habitat” is incorporated in the forest rights act 2006

2. For the first time in India, Baigas have been given habitat rights

3. Union Ministry of Environment Forest and Climate Change officially decides and declares Habitat

Rights for Primitive and Vulnerable Tribal Groups in any part of India
 

Which of the statements given above is/are correct?
 

(a) 1 and 2 only

(b) 2 and 3 only

(c) 3 only

(d) 1, 2 and 3

 

Answer (a)

 

  • The definition of ‘critical wildlife habitat’ is incorporated only in the Scheduled Tribes and Other Traditional Forest Dwellers (Recognition of Forest Rights) Act, 2006.

  • Union Ministry of Tribal Affairs is the nodal ministry and officially decides and declares Habitat Rights for Primitive and Vulnerable Tribal Groups in any part of India.

  • For the first time habitat rights have been given to Baigas under the Forest Rights Act of 2006.

  • Baiga tribals become India's first community to get habitat rights.

  • Critical wildlife habitats (CWH) are defined under the Forest Rights Act, 2006, as the “areas of national parks and sanctuaries where it has been specifically and clearly established, case by case, on the basis of scientific and objective criteria, that such areas are required to be kept as inviolate for the purposes of wildlife conservation.”

8. Consider the following
 

1. Birds

2. Dust blowing

3. Rain

4. Wind blowing
 

Which of the above spread plant diseases?
 

(a) 1 and 3 only

(b) 3 and 4 only

(c) 1, 2 and 4 only

(d) 1, 2, 3 and 4

 

Answer (d)

Nematodes, snails, birds, and wild and domestic animals often help dissemination of plant diseases.

The spores of many parasitic fungi are disseminated by air currents from diseased to disease-free host Pathogens like, bacteria are often disseminated by splashing of raindrops, as in case of Citrus canker disease.

Soil and field operation also disseminate the diseases as they result in dust blowing.

9. How is the National Green Tribunal (NGT) different from the Central Pollution Control Board (CPCB)?
 

1. The NGT has been established by an Act whereas the CPCB has been created by an executive order of the government

2. The NGT provides environmental justice and helps reduce the burden of litigation in the higher courts whereas the CPCB promotes cleanliness of streams and wells, and aims to improve the quality of air in the country
 

Which of the statements given above is/are correct?
 

a) 1 only

b) 2 only

c) Both 1 and 2

d) Neither 1 nor 2

 

Answer (b)

The National Green Tribunal has been established on 18.10.2010 under the National Green Tribunal Act 2010 for effective and expeditious disposal of cases relating to environmental protection and conservation of forests and other natural resources including enforcement of any legal right relating to environment and giving relief and compensation for damages to persons and property and for matters connected therewith or incidental thereto. 

The Tribunal’s dedicated jurisdiction in environmental matters shall provide speedy environmental justice and help reduce the burden of litigation in the higher courts. The Tribunal is mandated to make and endeavour for disposal of applications or appeals finally within 6 months of filing of the same.

The Central Pollution Control Board (CPCB), statutory organisation, was constituted in September, 1974 under the Water (Prevention and Control of Pollution) Act, 1974. Further, CPCB was entrusted with the powers and functions under the Air (Prevention and Control of Pollution) Act, 1981.

Functions of the CPCB, as spelt out in the Water (Prevention and Control of Pollution) Act, 1974, and the Air (Prevention and Control of Pollution) Act, 1981, (i) to promote cleanliness of streams and wells in different areas of the States by prevention, control and abatement of water pollution, and (ii) to improve the quality of air and to prevent, control or abate air pollution in the country.

Hence, statement 1 is incorrect and statement 2 is correct.

10. With reference to the ‘Global Alliance for Climate-Smart Agriculture’ (GACSA) which of the following statements is/are correct?
 

1. GACSA is an outcome of the climate summit held in Paris in 2015

2. Membership of GACSA does not create any binding obligations

3. India was instrumental in the creation of GACSA
 

Select the correct answer using the code given below
 

(a) 1 and 3 only

(b) 2 only

(c) 2 and 3 only

(d) 1, 2 and 3

 

Answer (b)

Statement 1 is incorrect because the concept of Climate-Smart Agriculture (CSA) was originally developed by FAO and officially presented and at the Hague Conference on Agriculture, Food Security and Climate Change in 2010, through the paper “Climate-Smart Agriculture: Policies, Practices and Financing for Food Security, Adaptation and Mitigation”. In 2014 an alliance was set up with this issue as its focal point: the GASCA (Global Alliance for Climate-Smart Agriculture).

GACSA is an inclusive, voluntary and action-oriented multi-stakeholder platform on Climate-Smart Agriculture (CSA). Its vision is to improve food security, nutrition and resilience in the face of climate change. GACSA aims to catalyse and help create transformational partnerships to encourage actions that reflect an integrated approach to the three pillars of CSA.

Statement 2 is correct because membership in the Alliance does not create any binding obligations and each member individually determines the nature of its participation.

Statement 3 is incorrect because India is just a signatory.

11. “Momentum for Change: Climate Neutral Now” is an initiative launched by
 

(a) The Intergovernmental Panel on climate change

(b) The UNEP Secretariat

(c) The UNFCCC Secretariat

(d) The World Meteorological Organization

 

Answer (c)
 

  • The UNFCCC secretariat launched its Climate Neutral Now initiative in 2015.

  • It is an initiative launched to urge individuals, companies, and governments to measure their climate footprint, reduce their greenhouse gas emissions as much as possible.

  • The secretariat in 2016 launched a new pillar under its Momentum for Change initiative focused on Climate Neutral Now, as part of larger efforts to showcase successful climate action around the world.

  • Climate neutrality is a three-step process, which requires individuals, companies, and governments to:

    • Measure their climate footprint;

    • Reduce their emissions as much as possible;

    • Offset what they cannot reduce with UN certified emission reductions.

UPSC 2019

1. Which one of the following groups of plants were domesticated in the ‘New World’ and introduced into the ‘Old World’?
 

(a) Tobacco, cocoa and rubber

(b) Tobacco, cotton and rubber

(c) Cotton, coffee and sugarcane

(d) Rubber, coffee and wheat

 

Answer (a)

Tobacco, cocoa, and rubber came to India through Europeans in the late medieval or early modern era and so these are the groups of plants that were domesticated in the 'New World' and introduced into the 'Old World'.

  • All of them originated in South America.

  • This process is called a Columbian exchange (named for Christopher Columbus).

  • It was the widespread transfer of plants, animals, culture, human populations, technology, diseases, and ideas between the Americas, West Africa, and the Old World during the 15th and 16th centuries.

Sugar cane is also the oldest crop and is introduced to India by Austronesian traders.

Rubber was native to Brazil.

Cotton and wheat were cultivated in India since the ancient era. Even there was evidence of Cotton and wheat since the Harappan era.

2. Which one of the following National Parks lies completely in the temperate alpine zone?
 

(a) Manas National Park

(b) Namdapha National Park

(c) Neora Valley National Park

(d) Valley of Flowers National Park
 

Answer:  (d)

Valley of flowers is famous for its alpine meadows. It wholly lies in temperate alpine zone. The valley has three sub-alpine between 3,200m and 3,500m which is the limit for trees, lower alpine between 3,500m and 3,700m, and higher alpine above 3,700m.

Consider the following statements:
 

  1. Asiatic lion is naturally found in India only.

  2. Double-humped camel is naturally found in India only.

  3. One-horned rhinoceros is naturally found in India only.
     

Which of the statements given above is / are correct?
 

(a) 1 only

(b) 2 only

(c) 1 and 3 only

(d) 1, 2 and 3
 

Answer: (a)

Statement 1: The Asiatic lion’s range is restricted to the Gir National Park and environs in the Indian state of Gujarat.

Statement 2: The Bactrian camel (Camelus bactrianus) is a large, even-toed ungulate native to the steppes of Central Asia.

Statement 3: The one-horned rhinoceros is native to the Indian subcontinent (not only India). The Indian rhinoceros once ranged throughout the entire stretch of the Indo-Gangetic Plain, but excessive hunting and agricultural development reduced their range drastically to 11 sites in northern India and southern Nepal.

In the context of which of the following do some scientists suggest the use of cirrus cloud thinning technique and the injection of sulphate aerosol into stratosphere?
 

(a) Creating the artificial rains in some regions

(b) Reducing the frequency and intensity of tropical cyclones

(c) Reducing the adverse effects of solar wind on the Earth

(d) Reducing the global warming
 

Answer: (d)

The ability of stratospheric sulfate aerosols to create a global dimming effect has made them a possible candidate for use in solar radiation management climate engineering projects to limit the effect and impact of climate change due to rising levels of greenhouse gases. Delivery of precursor sulfide gases such as sulfuric acid, hydrogen sulfide (H2S) or sulfur dioxide (SO2) by artillery, aircraft and balloons has been proposed.

Cirrus cloud thinning is a proposed form of climate engineering. Cirrus clouds are high cold ice that, like other clouds, both reflect sunlight and absorb warming infrared radiation. However, they differ from other types of clouds in that, on average, infrared absorption outweighs sunlight reflection, resulting in a net warming effect on the climate. Therefore, thinning or removing these clouds would reduce their heat trapping capacity, resulting in a cooling effect on Earth’s climate.

Which of the following are in Agasthyamala Biosphere Reserve?
 

(a) Neyyar, Peppara and Shendurney Wildlife Sanctuaries; and Kalakad Mundanthurai Tiger Reserve

(b) Mudumalai, Sathyamangalam and Wayanad Wildlife Sanctuaries; and Silent Valley National Park

(c) Kaundinya, Gundla Brahme-swaram and Papikonda Wildlife Sanctuaries; and Mukurthi National Park

(d) Kawal and Sri Venkateswara Wildlife Sanctuaries; and Nagarjunasagar-Srisailam Tiger Reserve
 

Answer: (a)
Agasthyamala Biosphere Reserve is a unique genetic reservoir of cultivated plants.

It is located in the Western Ghats in the southern India.

 Three wildlife sanctuaries, Shendurney, Peppara and Neyyar, are located in the site, as well as the Kalakad Mundanthurai Tiger reserve.

Consider the following statements:
 

  1. Some species of turtles are herbivores.

  2. Some species of fish are herbivores.

  3. Some species of marine mammals are herbivores.

  4. Some species of snakes are viviparous.
     

Which of the statements given above are correct?
 

(a) 1 and 3 only

(b) 2, 3 and 4 only

(e) 2 and 4 only

(d) 1, 2, 3 and 4
 

Answer: (d)

Statement 1: Sea turtles may be carnivorous (meat eating), herbivorous (plant eating), or omnivorous (eating both meat and plants). The jaw structure of many species indicates their diet.

Some species change eating habits as they age. For example, green sea turtles are mainly carnivorous from hatching until juvenile size; then progressively shift to an herbivorous diet. See https://seaworld.org/animals/all-about/sea-turtles/diet/

Statement 2: Herbivorous fishes are fishes that eat plant material. Surgeonfish and parrotfish are two familiar MAR examples, often seen browsing and scraping on reef algae. See http://www.healthyreefs.org/cms/healthy-reef-indicators/herbivorous-fish-abundance/

Statement 3: Marine herbivores are found within four groups of species in the animal kingdom — invertebrates, fish, reptiles and mammals — and include zooplankton, mollusks, the green sea turtle, the marine iguana and some fish species. Manatees and dugongs are the only herbivores among marine mammals. See https://animals.mom.me/marine-animals-herbivores-11740.html

Statement 4: Snakes are reptiles, and most species of snakes will follow the general rules of reptilian reproduction — the mother will lay a clutch of eggs that will hatch into baby snakes. Such snakes are referred to as oviparous. A few exceptional species of snakes diverge from this general rule. They are referred to as viviparous, and they give birth to live young. Both oviparous and viviparous snakes have uteri, or wombs; however, only viviparous snakes form placental attachments between their uteri and the developing fetuses. See https://animals.mom.me/snakes-wombs-10778.html

Why is there a great concern about the ‘microbeads’ that are released into environment?
 

(a) They are considered harmful to marine ecosystems.

(b) They are considered to cause skin cancer in children.

(c) They are small enough to be absorbed by crop plants in irrigated fields.

(d) They are often found to be used as food adulterants.
 

Answer: (a)

Microbeads are manufactured solid plastic particles of less than one millimeter in their largest dimension. They are most frequently made of polyethylene but can be of other petrochemical plastics such as polypropylene and polystyrene. They are used in exfoliating personal care products, toothpastes and in biomedical and health-science research.

Microbeads can cause plastic particle water pollution and pose an environmental hazard for aquatic animals in freshwater and ocean water.

For the measurement/estimation of which of the following are satellite images/remote sensing data used?
 

  1. Chlorophyll content in the vegetation of a specific location

  2. Greenhouse gas emissions from rice paddies of a specific location

  3. Land surface temperatures of a specific location
     

Select the correct answer using the code given below.
 

(a) 1 only

(b) 2 and 3 only

(c) 3 only

(d) 1, 2 and 3
 

Answer: (d)

Statement 1: Thematic mappers take images in multiple wavelengths of electro-magnetic radiation (multi-spectral) and based on this survey – maps of land cover and land use from thematic mapping can be used to prospect for minerals, detect or monitor land usage, detect invasive vegetation, deforestation, and examine the health of indigenous plants and crops, including entire farming regions or forests. See https://en.wikipedia.org/wiki/Remote_sensing#Applications_of_remote_sensing

Statement 2: Radiometers and photometers are the most common instrument in use, collecting reflected and emitted radiation in a wide range of frequencies. They may also be used to detect the emission spectra of various chemicals, providing data on chemical concentrations in the atmosphere.

Statement 3:  It is possible to measure land surface temperatures, see here https://rmets.onlinelibrary.wiley.com/doi/pdf/10.1002/met.287

Which of the following statements are correct about the deposits of ‘methane hydrate?
 

  1. Global warming might trigger the release of methane gas from these deposits.

  2. Large deposits of ‘methane hydrate’ are found in Arctic Tundra and under the seafloor.

  3. Methane in atmosphere oxidizes to carbon dioxide after a decade or two.
     

Select the correct answer using the code given below.
 

(a) 1 and 2 only

(b) 2 and 3 only

(c) 1 and 3 only

(d) 1, 2 and 3
 

Answer: (d)

Statement 1 and Statement 2: Large but poorly known amounts of methane are trapped in the sediments beneath the sea floor (in Tundra region and elsewhere), frozen into a form of water ice called methane hydrate.

At low temperatures the methane hydrates on the sea floor are stable, but if the water and the sea floor become warmer, then the hydrates can break down. Because microorganisms then oxidize the resulting methane gas to form the greenhouse gas carbon dioxide (CO2), methane hydrates have recently become a topic of intense discussion within the context of climate change. See https://worldoceanreview.com/en/wor-1/energy/methane-hydrates/
 

Statement 3: Methane is relatively short-lived in the atmosphere; a molecule of methane is oxidized to water and carbon dioxide within a decade or so, mainly by reaction with another trace gas, the hydroxyl radical OH-. Thus, unlike the case of carbon dioxide (which stays in the atmosphere longer than methane), a concerted effort to reduce methane emissions would have almost immediate results in terms of reduction of greenhouse effect. See http://earthguide.ucsd.edu/virtualmuseum/climatechange1/03_3.shtml  

Consider the following:

  1. Carbon monoxide

  2. Methane

  3. Ozone

  4. Sulphur dioxide
     

Which of the above are released into atmosphere due to the burning of crop/biomass residue?
 

(a) 1 and 2 only

(b) 2, 3 and 4 only

(c) 1 and 4 ‘only

(d) 1, 2, 3 and 4
 

Answer: (d)

Agricultural crop residue burning contribute towards the emission of greenhouse gases (CO2, N2O, CH4), air pollutants (CO, NH3, NOx, SO2, NMHC, volatile organic compounds), particulates matter and smoke thereby posing threat to human health.

Satellite observations have revealed elevated levels of O3 (Ozone), CO and aerosols over vast areas of Central Africa and South America, over the tropical Atlantic, and the Indian Ocean due to long-range transport of pollutants emitted from biomass burning.

See http://www.aaqr.org/files/article/619/40_AAQR-13-01-OA-0031_422-430.pdf

In India, the use of carbofuran, methyl parathion, phorate and triazophos is viewed with apprehension. These chemicals are used as
 

(a) pesticides in agriculture

(b) preservatives in processed foods

(c) fruit-ripening agents

(d) moisturising agents in cosmetics
 

Answer: (a)

Justification: Carbofuran is one of the most toxic carbamate pesticides. It is marketed under the trade names Furadan, by FMC Corporation and Curater, among several others. It is used to control insects in a wide variety of field crops, including potatoes, corn and soybeans.

Excerpts from a news report:

“CARDAMOM Planters Marketing Co-operative Society in Kerala has appealed to the high court to suspend the ban on the use of extremely toxic and highly toxic pesticides.

To promote organic farming in Kerala, the state agriculture department had ordered a ban on the use of these two categories of pesticides on May 7. The order was to be implemented within 10 days.

During this period the Kerala Agriculture University was asked to provide alternatives to the banned pesticides, which include carbofuran, phorate, methyl parathion, monocrotophos, methyl demethon, prophenophos and triazophos. The university suggested less hazardous pesticides, like acephate, carbaryl, dimethoate and flubendiamide.”

See https://www.downtoearth.org.in/news/pesticide-ban-lands-kerala-in-court-33657

Consider the following statements:

  1. Under Ramsar Convention, it is mandatory on the part of the Government of India to protect and conserve all the wetlands in the territory of India.

  2. The Wetlands (Conservation and Management) Rules, 2010 were framed by the Government of India based on the recommendations of Ramsar Convention.

  3. The Wetlands (Conservation and Management) Rules, 2010 also encompass the drainage area or catchment regions of the wetlands as determined by the authority.
     

Which of the statements given above is / are correct?
 

(a) 1 and 2 only

(b) 2 and 3 only

(c) 3 only

(d) 1, 2 and 3
 

Answer: (b)

Statement 1: There is no such binding provision. Contracting Parties to the Ramsar convention make a commitment to:

  • designate at least one site that meets the Ramsar criteria for inclusion in the List of Wetlands of International Importance

  • promote the conservation and wise use of wetlands

  • include wetland conservation within their national land-use planning

  • establish nature reserves on wetlands and promote wetland training, and

  • consult with other Contracting Parties about the implementation of the Ramsar Convention.

See https://www.environment.gov.au/water/wetlands/ramsar

Statement 2: This is a correct statement.  

Clause 3 of 2010 rules: “Protected wetlands –

Based on the significance of the functions performed by the wetlands for overall well being of the people and for determining the extent and level of regulation, the following wetlands shall be regulated under these rules, namely:-

(i) wetlands categorised as Ramsar Wetlands of International Importance under the Ramsar Convention as specified in the Schedule;

See https://indiankanoon.org/doc/106740276/

Statement 3: As per the definitions stated under the rules, “wetland” means an area or of marsh, fen, peatland or water…..includes all inland waters such as lakes, reservoir, tanks, backwaters, lagoon, creeks, estuaries and manmade wetland and the zone of direct influence on wetlands that is to say the drainage area or catchment region of the wetlands as determined by the authority but does not include main river channels, paddy fields and the coastal wetland.

See https://indiankanoon.org/doc/106740276/

Consider the following statements:
 

  1. Agricultural soils release nitrogen oxides into environment.

  2. Cattle release ammonia into environment.

  3. Poultry industry releases reactive nitrogen compounds into environment.
     

Which of the statements given above is/are correct?
 

(a) 1 and 3 only

(b) 2 and 3 only

(c) 2 only

(d) 1, 2 and 3
 

Answer: (d)

Agricultural soils contributed to over 70% of N2O emissions from India in 2010, followed by waste water (12%) and residential and commercial activities (6%). Since 2002, N2O has replaced methane as the second largest Greenhouse Gas (GHG) from Indian agriculture.

Cattle account for 80% of the ammonia production, though their annual growth rate is 1%, due to a stable population.

The poultry industry, on the other hand, with an annual growth rate of 6%, recorded an excretion of reactive nitrogen compounds of 0.415 tonnes in 2016.

See https://www.thehindu.com/sci-tech/energy-and-environment/nitrogen-emissions-going-up-study/article24090131.ece

Why are dewdrops not formed on a cloudy night?
 

(a) Clouds absorb the radiation released from the Earth’s surface.

(b) Clouds reflect back the Earth’s radiation.

(c) The Earth’s surface would have low temperature on cloudy nights.

(d) Clouds deflect the blowing wind to ground level.
 

Answer: (b)

The dew formation is more when the sky is clear and less when it is cloudy. When the sky is clear and the trees and plants are cooler at nights, there is more evaporation of water and hence more dew formation. But when it is cloudy, trees and plants do not get cool in the night and hence there is less dew formation. As the sun raises high in the sky, these dew drops evaporate into

Consider the following statements:
 

  1. According to the Indian Patents Act, a biological process to create a seed can be patented in India.

  2. In India, there is no Intellectual Property Appellate Board.

  3. Plant varieties are not eligible to be patented in India.
     

Which of the statements given above is/are correct?
 

(a) 1 and 3 only

(b) 2 and 3 only

(c) 3 only

(d) 1, 2 and 3
 

Answer: (c)

 Statement 1: The SC recently ruled that one can claim patents on GM cotton seeds, in a case related to Monsanto. See https://www.downtoearth.org.in/blog/agriculture/was-there-a-victory-for-monsanto-in-india-s-supreme-court-on-a-patent-matter–62800   

This judgment overturned an order of the Delhi High Court which held that genetically modified plants, genetically modified seeds and gene sequences that provide genetic traits to plants are not patentable subject matter in India.

The decision comes in the ongoing dispute between the Monsanto Group of Companies and the Nuziveedu Group of Companies.

Statement 2 is wrong.

Statement 3: Article 3(j) of the India’s patent Act excludes from patentability “plants and animals in whole or in any part thereof other than microorganisms  but including seeds, varieties, and species, and essentially biological processes for production or propagation of plants and animals”.

See https://www.ifoam.bio/en/news/2018/05/08/indian-supreme-court-says-seeds-plants-and-animals-are-not-patentable

The word ‘Denisovan’ is sometimes mentioned in media in reference to
 

(a) fossils of a kind of dinosaurs

(b) an early human species

(c) a cave system found in North-East India.

(d) a geological period in the history of Indian subcontinent
 

Answer: (b)

The Denisovans or Denisova hominins are an extinct species or subspecies of archaic humans in the genus Homo.

Scientists have uncovered the most complete remains yet from the mysterious ancient-hominin group known as the Denisovans.

The jawbone, discovered high on the Tibetan Plateau and dated to more than 160,000 years ago, is also the first Denisovan specimen found outside the Siberian cave in which the hominin was uncovered a decade ago — confirming suspicions that Denisovans were more widespread than the fossil record currently suggests.

The research marks the first time an ancient human has been identified solely through the analysis of proteins. With no usable DNA, scientists examined proteins in the specimen’s teeth, raising hopes that more fossils could be identified even when DNA is not preserved.

See https://www.nature.com/articles/d41586-019-01395-0

Consider the following statements:
 

The Environment Protection Act, 1986 empowers the Government of India to

  1. state the requirement of public participation in the process of environmental protection, and the procedure and manner in which it is sought

  2. lay down the standards for emission or discharge of environmental pollutants from various sources
     

Which of the statements given above is/ are correct?
 

(a) 1 only

(b) 2 only

(c) Both 1 and 2

(d) Neither 1 nor 2
 

Answer: (b)

EPA, 1986 doesn’t contain any provision related to public participation. Section 6 of the Act authorises that the Central Govt regulates pollution control norms.

See http://bch.cbd.int/database/attachment/?id=19052

Consider the following statements:
 

  1. As per recent amendment to the Indian Forest Act, 1927, forest dwellers have the right to fell the bamboos grown on forest areas.

  2. As per the Scheduled Tribes and Other Traditional Forest Dwellers (Recognition of Forest Rights) Act, 2006, bamboo is a minor forest produce.

  3. The Scheduled Tribes and Other Traditional Forest Dwellers (Recognition of Forest Rights) Act, 2006 allows ownership of minor forest produce to forest dwellers.
     

Which of the statements given above is / are correct?
 

(a) 1 and 2 only

(b) 2 and 3 only

(c) 3 only

(d) 1, 2 and 3
 

Answer: (b)

Statement 1: Under the original Act, the definition of tree includes palms, bamboos, stumps, brush-wood, and canes.  The amendment act amends this definition of tree to remove the word bamboos.

Since bamboo is defined as a tree under the Act, its inter-state movement requires permit when in transit in other states. Consequent to the amendment, felling or transportation of bamboos growing in non-forest areas will not require any permits.

See https://www.prsindia.org/billtrack/indian-forest-amendment-bill-2017

Statement 2: The Act recognises bamboo as an MFP and vests the “right of ownership, access to collect, use and dispose of minor forest pr oduce” with Scheduled Tribes and traditional forest dwellers

See https://www.downtoearth.org.in/news/bamboo-now-a-minor-forest-produce-33239

Statement 3: Section 3(1) of the act provides a right of ownership, access to collect, use, and dispose of minor forest produce( includes all non-timber forest produce of plant origin) which has been traditionally collected within or outside village boundaries.

Recently, there was a growing awareness in our country about the importance of Himalayan nettle (Girardinia diversifolia) because it is found to be a sustainable source of
 

(a) anti-malarial drug

(b) bio-diesel

(c) pulp for paper industry

(d) textile fibre
 

Answer: (d)

Girardinia diversifolia (Himalayan nettle), a fibre-yielding plant, has become an important livelihood option for people living in the remote mountainous villages of the Hindu Kush Himalaya.

There is a community in Khar, a hamlet in Darchula district in far-western Nepal, which produces fabrics from Himalayan nettle. The fabric and the things made from it are sold in local as well as national and international markets as high-end products.

See https://www.downtoearth.org.in/blog/environment/khar-s-experimentation-with-himalayan-nettle-brings-recognition-57880

In the context of proposals to the use of hydrogen-enriched CNG (H-CNG) as fuel for buses in public transport, consider the following statements:
 

  1. The main advantage of the use of H-CNG is the elimination of carbon monoxide emissions.

  2. H-CNG as fuel reduces carbon dioxide and hydrocarbon emissions.

  3. Hydrogen up to one-fifth by volume can be blended with CNG as fuel for buses.

  4. H-CNG makes the fuel less expensive than CNG.
     

Which of the statements given above is / are correct?
 

(a) 1 only

(b) 2 and 3 only

(c) 4 only

(d) 1, 2, 3 and 4
 

Answer: (b)

HCNG is a mixture of compressed natural gas (CNG) and some % Hydrogen by energy.

HCNG which may be used as a fuel of Internal Combustion Engine (ICE) is considered a cleaner source of fuel, more powerful and offers more mileage then even CNG.
 

Some advantages of HCNG:

  • HCNG reduces emissions of CO up to 70%. S1 is incorrect since it is not possible.

  • Enables up to 5 % savings in fuel.

  • Better performance due to higher Octane rating of H2.

S3: Delhi Government has tied up with Indian Oil Corporation Limited to study the technology and infrastructure needs to induct 50 HCNG buses on a trial basis. IOCL has plans to mix (18-20) % Hydrogen in these buses.

S4: Current cost of H2 is more than the cost of Natural Gas. So, HCNG’s cost is more than CNG.

See https://www.insightsonindia.com/2018/09/11/rajya-sabha-tv-in-depth-hcng-fuel-of-the-future/

UPSC 2019

1. In the context of the recent advances in human reproductive technology, Pronuclear transfer technology is used for:
 

(a) Fertilization of egg into vitro by the donor sperm

(b) Genetic modification of sperm producing cells

(c) Development of stem cells into functional embryos

(d) Prevention of mitochondrial diseases in offspring
 

Answer: (d)

In pronuclear transfer, the mother’s egg is first fertilized with the father’s sperm, producing a zygote. The pronuclei of the egg and sperm are then removed from the zygote and inserted into a donor egg that has been fertilized and has had its own nucleus removed (a pronucleus is the nucleus of the egg or sperm at the stage of fertilization prior to nucleus fusion). The zygote derived from the donor egg is then implanted into the mother’s uterus. It is NOT fully in vitro.

Option D is correct, this is a new strategy to prevent transmission of mitochondrial diseases in offspring, see https://academic.oup.com/hmg/article/20/R2/R168/640414

 2. With reference to the carbon nanotubes, consider the following statements:
 

  1. They can be used as carriers of drugs and antigens in the human body.

  2. They can be made into artificial blood capillaries for an injured part of human body.

  3. They can be used in biochemical sensors.

  4. Carbon nanotubes are biodegradable
     

Which of the statements given above are correct?

(a) 1 and 2 only

(b) 2, 3 and 4 only

(c) 1, 3 and 4 only

(d) 1, 2, 3, and 4

Answer: (d)

Carbon nanotubes - 
These are cylindrical molecules that consist of rolled-up sheets of single-layer carbon atoms (graphene). 
There are single-walled nanotubes (with a diameter of less than 1 nanometer) and multi-walled nanotubes (with diameters reaching more than 100 nm). 
These are chemically bonded with sp2 bonds which is an extremely strong form of molecular interaction. 
They develop ultra-high-strength and low-weight materials that possess highly conductive electrical and thermal properties. 

Carbon nanotubes are promising drug delivery platforms that can be functionalized with a variety of biomolecules, such as antibodies, proteins, antigens, and DNA. 

Carbon nanotubes can easily penetrate cells and improve the pharmacological and therapeutic profile, and efficacy of the drug. Hence statement 1 is correct. 

The biotechnology facilitates the making of nanodevices using blood-compatible nanomaterials as building blocks for biomedical applications such as artificial implants including structural tissue replacements, artificial blood vessels, or functional devices such as drug delivery matrices. Hence statement 2 is correct. 

NASA has successfully demonstrated a miniaturized electronics technology with extremely high sensitivity for in-vitro detecting specific biomarker signatures, which is based on incorporating embedded vertically aligned carbon nanotubes as nanoelectrode arrays in diagnostics devices. Hence statement 3 is correct. 

It has been demonstrated that functionalised carbon nanotubes can be degraded by oxidative enzymes. Multiple types of microbes including bacteria and fungi have the ability to degrade carbon nanotubes (CNTs), graphene (GRA), and their derivatives. Hence statement 4 is correct.

3. Consider the following activities :
 

  1. Spraying pesticides on a crop field

  2. Inspecting the craters of an active volcano

  3. Collecting breath samples from spouting whales for DNA analysis
     

At the present level of technology, which of the above activities can be successfully carried out using drones?
 

(a) 1 and 2

(b) 2 and 3

(c) 1 and 3

(d)1, 2 and 3
 

Answer: (d)

Statement 1: This is widely used in developed countries to save manpower.

Statement 2:  Drones can go where volcanologists can’t, e.g. craters of volcanoes, giving researchers access to potentially life-saving data. 

Statement 3: Correct. Source: https://www.theverge.com/2017/10/13/16468698/drone-humpback-whale-microbiome-health-conservation

Scientists flew a small drone over the blowhole of a few humpback whales in the US and Canada to collect the microbes living inside their breath. Sampling the community of microbes and bacteria living inside whales, called the microbiome, can help us better understand what makes a healthy whale, and what happens when a whale gets sick.

4. Consider the following statements:
 

  1. Genetic changes can be introduced in the cells that produce eggs or sperms of a prospective parent

  2. A person’s genome can be edited before birth at the early embryonic stage

  3. Human induced pluripotent stem cells can be injected into the embryo of a pig.
     

Which of the statements given above is/are correct?
 

(a) 1 only

(b) 2 and 3 only

(c) 2 only

(d) 1, 2 and 3
 

Answer: (d)

Statement 1: The DNA in any cell can be altered through environmental exposure to certain chemicals, ultraviolet radiation, other genetic insults, or even errors that occur during the process of replication. If a mutation occurs in a germ-line cell (one that will give rise to gametes, i.e., egg or sperm cells), then this mutation can be passed to an organism’s offspring. This means that every cell in the developing embryo will carry the mutation. As opposed to germ-line mutations, somatic mutations occur in cells found elsewhere in an organism’s body. Such mutations are passed to daughter cells during the process of mitosis, but they are not passed to offspring conceived via sexual reproduction.
 

Statement 2: This is correct, see news at MIT review https://www.technologyreview.com/2018/11/25/138962/exclusive-chinese-scientists-are-creating-crispr-babies/ ). This was controversially achieved by a team at the Southern University of Science and Technology, in Shenzhen, which recruited couples in an effort to create the first gene-edited babies. They planned to eliminate a gene called CCR5 in hopes of rendering the offspring resistant to HIV, smallpox, and cholera. Also see this article https://www.nature.com/articles/d41586-019-00673-1 and this research piece https://www.ncbi.nlm.nih.gov/pmc/articles/PMC6813942/
 

Statement 3: This is correct as well, see https://europepmc.org/article/pmc/3883495

Domesticated animals such as dogs, pigs and cows are considered excellent models for long-term experiments in regenerative medicines, and biomedical research in general, because of their similarities in physiology with humans compared to the laboratory mouse or rat.

So, the techniques used to generate induced pluripotent stem cells from mouse and human were also suitable in pigs.

5. Which of the following statements are correct regarding the general difference between plant cells and animal cells?
 

  1. Plant cells have cellulose cell walls whilst animal cells do not.

  2. Plant cells do not have plasma membrane unlike animals cells which do

  3. Mature plant cell has one large vacuole whilst animal cell has many small vacuoles
     

Select the correct answer using the given code below-
 

(a) 1 and 2 only

(b) 2 and 3 only

(c) 1 and 3 only

(d) 1, 2 and 3
 

Answer: (c)

Statement 1 and Statement 2: Cells are the basic unit of a living organism and where all life processes are carried out.

Animal cells and plant cells share the common components of a nucleus, cytoplasm, mitochondria and a cell membrane.

Plant cells have three extra components, a vacuole, chloroplast and a cell wall.

Statement 3: Vacuoles are storage bubbles found in cells. They are found in both animal and plant cells but are much larger in plant cells. Vacuoles might store food or any variety of nutrients a cell might need to survive.

Source: https://www.bbc.co.uk/bitesize/topics/znyycdm/articles/zmrtng8

6. Which of the following are the reasons/factors for exposure to benzene pollution?
 

  1. Automobile exhaust

  2. Tobacco smoke

  3. Wood burning

  4. Using varnished wooden furniture

  5. Using products made of polyurethane
     

Select the correct answer using the given code below-
 

(a) 1, 2 and 3 only

(b) 2 and 4 only

(c) 1, 3 and 4 only

(d) 1, 2, 3, 4 and 5 only
 

Answer: (d)

Statement 1: This is obvious.

Statement 2: Benzene is a by-product of the combustion of tobacco in cigarettes.

Statement 3: Wood smoke was the biggest contributor of many organic compounds, including benzene, ethene and ethyne, all of which are known to be harmful to human health. Up to 70 per cent of benzene detected in the air in Northern Europe was from wood smoke. See https://ec.europa.eu/environment/integration/research/newsalert/pdf/4si4_en.pdf

Statement 4: Benzene, ethylene glycol or formaldehyde could be released from varnished wooden furniture or new carpets.

Statement 5: This is correct, see http://gpcpe.org/wp-content/uploads/2019/03/Plastic-and-Health-The-Hidden-Costs-of-a-Plastic-Planet-February-2019.pdf Polyurethane (PUR and PU) is a polymer composed of organic units joined by carbamate (urethane) links. While most polyurethanes are thermosetting polymers that do not melt when heated, thermoplastic polyurethanes are also available.

7. With reference to Indian Elephants, consider the following statements :
 

1. The leader of an elephant group is a female.
2. The maximum gestation period can be 22 months.
3. An elephant can normally go on calving till the age of 40 years only.
4. Among the States in India, the highest population is in Kerala.
 

Which of the statements given above is/are correct ?
 

(a) 1and 2 only

(b) 2 and 4 only

(c) 3 only

(d) 1,3 and 4 only.
 

Answer: (a)

The leader of an elephant group is the oldest female. She leads the group in search of food and water. Hence statement 1 is true. 
The gestation period of the breeding pair is nearly 3 weeks or 22 days which yields a single baby. During birth, females of the herd surround the mother for her protection. Hence statement 2 is correct. 
The average life expectancy of an elephant is 70 years and at the age of 18 years, its adulthood starts. Hence statement 3 is incorrect. 
South Indian states dominate in having a number of elephants. Among the south Indian states, Karnataka leads the table with 6,049 elephants followed by Kerala. Hence statement 4 is incorrect. 

Additional Information 
Project Elephant is a Central Government sponsored scheme launched in February 1992. It is implemented in 16 out of 29 states of the country. 
World Elephant Day is celebrated on 12th August every year. 
There are around 28,000 elephants in India with around 25% of them in Karnataka.

8. Which of the following Protected areas are located in Cauvery basin?
 

1. Nagarhole National Park.
2. Papikonda National Park.
3. Sathyamangalam Tiger Reserve
4. Wayanad Wildlife Sanctuary
 

Select the correct answer using the code given below:
 

(a) 1 and 2 only
(b) 3 and 4 only
(c) 1, 3 and 4 only
(d) 1,2,3 and 4
 

Answer: (c)

All except 2 are a part of Cauvery basin. Papikonda remains entirely inside the East and West Godavari districts near Godavari basin after 2014 and after the construction of Polavaram Dam.

9. With reference to India’s Biodiversity, Ceylon frogmouth, Coppersmith barbet, Gray-chinned minivet and White-throated redstart are
 

(a) Birds

(b) Primates

(c) Reptiles

(d) Amphibians
 

Answer: (a)

Ceylon Frogmouth is also known as Srilanka frogmouth. It is a bird species found in the Western Ghats. Its IUCN status is the least concern. Its biological name is Batrachostomus moniliger. 
Coppersmith Barbet is also known as Crimson-breasted barbet is bird species found in the whole Indian Subcontinent. Its IUCN status is the least concerned. Its scientific name is Psilopogon haemacephalus. 
Grey Chinned minivet is bird species found from the Himalayas to China. Its IUCN status is least concerned. Pericrocotus Solaris is its biological name. 
White-throated redstart is a bird species found in India, Nepal, Bhutan, and China. It is also given the status of least concern by IUCN. Its biological name is Phoenicurus schisticeps.

10. Which one of the following protected areas is well known for the conservation of a sub-species of the Indian swamp deer (Barasingha) that thrives well on hard ground and is exclusively graminivorous?
 

(a) Kanha National Park

(b) Manas National Park

(c) Mudumalai Wildlife Sanctuary

(d) Tal Chhapar Wildlife Sanctuary
 

Answer: (a)

Barasingha is the one of the rarest species of deer family spread across central and northern India only in small congregations. The deer is native to India and Nepal. Barasingha are also known as swamp deer due to their nature to survive close to river valleys or swampy regions. Kanha National Park is located in Madhya Pradesh and a renowned place for tigers. But the park’s major achievement is to save nearly extinct barasingha (swamps deers). It was a landmark achievement by preserve the highly endangered barasingha. Once it only 66 of these in Kanha, but careful BarahsIngha conservation and management raised their population to over 500.

11. Which of the following are the most likely places to find the musk deer in its natural habitat?

  1. Askot Wildlife Sanctuary

  2. Gangotri national Park

  3. Kishanpur Wildlife Sanctuary

  4. Manas national Park
     

Select the correct answer using the given code below
 

(a) 1 and 2 only

(b) 2 and 3 only

(c) 3 and 4 only

(d) 1 and 4 only
 

Answer: (a)

Statement 1: Askot Musk Deer Sanctuary is located 54 km from Pithoragarh near the town of Askot in Uttarakhand. As the name suggests, the sanctuary has been set up primarily for the conservation of musk deer and its natural habitat.

Statement 2: Various rare and endangered species like bharal or blue sheep, black bear, brown bear, Himalayan Monal, Himalayan Snowcock, Himalayan Thar, musk deer and Snow leopard are found in the park.

Statement 3 and Statement 4: Musk deer are largely distributed in the Himalayan high altitude belts, so Manas and Kishanpur can be discounted.

12. In rural road construction, the use of which of the following is preferred for ensuring environmental sustainability or to reduce carbon footprint?
 

  1. Copper slag

  2. Cold mix asphalt technology

  3. Geotextiles

  4. Hot mix asphalt technology

  5. Portland cement
     

Select the correct answer using the given code below
 

(a) 1, 2 and 3 only

(b) 2, 3 and 4 only

(c) 4 and 5 only

(d) 1 and 5 only
 

Answer: (a)
The use of copper slag in cement and concrete provides potential environmental as well as economic benefits for all related industries. Particularly in areas where a considerable amount of copper slag is produced. Hence, statement 1 is correct. 

Production of cold asphalt mix does not require high investment in equipment. This makes it economically relatively pollution-free (no objectionable fumes or odors). Cold asphalt mixes can be used both for initial construction (100% virgin mixes) and for recycling of asphalt pavements. Hence, statement 2 is correct. 

Geotextiles are mostly used in road construction, especially to fill gaps between the roads to improve soil structure. Geotextile makes poor soil more beneficial for use and then easy to build in difficult places also. It helps to prevent the erosion of soil but allows the water to drain off. Hence, statement 3 is correct. 

Portland cement and hot asphalt mix are the major CO2 emitters. Hot Mix Asphalt (HMA) is a combination of approximately 95% stone, sand, or gravel bound together by asphalt cement, a product of crude oil. Hence, statements 4 and 5 are incorrect.

13. Steel slag can be the material for which of the following?
 

  1. Construction of base road

  2. Improvement of agriculture soil

  3. Production of cement
     

Select the correct answer using the given code below
 

(a) 1 and 2 only

(b) 2 and 3 only

(c) 1 and 3 only

(d) 1, 2 and 3
 

Answer: (d)

Steel slag, a by-product of steel making, is produced during the separation of the molten steel from impurities in steel-making furnaces. One of the ingredients of asphalt is steel slag, a by-product of the steel and iron production processes. Asphalt roads are made of a mixture of aggregates, binders, and fillers. The aggregates are typically iron or steel slag, sand, gravel, mixed with asphalt that produces bitumen. The bitumen, produced by the refineries, plays an important role in road construction. Hence, statement 1 is correct. 
 

Steel slags can be used in several activities, such as construction and paving, and also in the agricultural sector Steel slag has the ability to correct soil acidity, as it contains some nutrients for the plants and also as silicate fertilizer that is capable of providing silicon to the plants. Hence, statement 2 is correct. 
 

Steel slag, another waste from the Iron & Steel Industry, has shown potential for use as a raw mix component up to 10% in the manufacture of cement clinker. Steel slag can also replace granulated blast furnace slag up to 10% in the manufacture of Portland Slag Cement. Steel slag has been used successfully to treat acidic water discharges from abandoned mines. Hence, statement 3 is correct.

14. Consider the following statements
 

  1. Coal ash contains arsenic, lead and mercury.

  2. Coal fired power plants release sulphur dioxide and oxides of nitrogen into the environment.

  3. High ash content is observed in Indian Coal.
     

Which of the above given statements is/are correct?
 

(a) 1 only

(b) 2 and 3 only

(c) 3 only

(d) 1, 2 and 3
 

Answer: (d)

Coal itself isn't a particularly toxic material. But after it's burned for domestic or commercial purposes it produces ash. Ash includes lead, mercury, cadmium, chromium, arsenic, and selenium, etc. All in levels that may threaten human health. Hence, statement 1 is correct. 
 

Thermal power plants produce large amounts of nitrogen oxides and sulfur dioxide. The pollutants that cause acid rain—when they burn fossil fuels, especially coal, to produce energy. Hence, statement 2 is correct. 
 

India's domestic coal reserves have a high ash content of up to 40 to 45 percent. Hence, statement 3 is correct.

15. What is the use of Biochar in farming?
 

  1. Biochar can be used as a part of the growing medium in the vertical farming

  2. When biochar is a part of the growing medium, it promotes the growth of nitrogen fixing microorganisms.

  3. When biochar is a part of the growing medium, it enables the medium to retain water for a longer time.
     

Which of the above given statements is/are correct?
 

(a) 1 and 2 only

(b) 2 only

(c) 1 and 3 only

(d) 1, 2 and 3 only
 

Answer: (d)

Biochar is a stable, carbon–rich form of charcoal that is applied to soil.

Some biochars can increase soil fertility, water holding capacity and crop productivity.

Use of biochar in vertical farming (see the article https://www.thehindu.com/business/agri-business/biochar-compost-tea-are-the-new-recipe/article7830057.ece ) can increase output significantly.

Adding biochar to soil increases its carbon content and could help mitigate greenhouse gas emissions.

Biochar is shown to have increased abundance of mycorrhizal fungi, assisting nutrient uptake by plants.

16. If a particular plant species is placed under Schedule VI of the Wildlife Protection Act,1972, what is the implication?
 

(a) A licence is required to cultivate that plant.

(b) Such a plant cannot be cultivated under any circumstances.

(c) It is a Genetically Modified crop plant.

(d) Such a plant is invasive and harmful to the ecosystem.
 

Answer: (a)

These are Endemic plants where Cultivation of specified plants without licence is prohibited.

From the Wildlife protection Act: (1) No person shall cultivate a specified plant except under and in accordance with a licence granted by the Chief Wild Life Warden or any other officer authorised by the State Government in this behalf: Provided that nothing in this section shall prevent a person, who immediately before the commencement of the Wild Life (Protection) (Amendment) Act, 1991 (44 of 1991), was cultivating a specified plant from carrying on such cultivation for a period of six months from such commencement or where he has made an application within that period for the grant of a licence to him, until the licence is granted to him or he is informed in writing that a licence cannot be granted to him.

See Chapter IIIA 17C http://legislative.gov.in/sites/default/files/A1972-53_0.pdf

17. What is/are the advantage/advantages of zero tillage in agriculture ?
 

1. Sowing of wheat is possible without burning the residue of the previous crop.

2. Without the need for nursery of rice saplings, direct planting of paddy seeds in the wet soil is possible.

3. Carbon sequestration in the soil is possible.
 

Select the correct answer using the code given below:
 

(a) 1 and 2 only

(b) 2 and 3 only

(c) 3 only

(d) 1,2 and 3 only
 

Answer: (d)

Tillage is an agriculture land preparation through mechanical agitation which includes digging, stirring and overturning.

Statement 1: Zero tillage is the process where the crop seed will be sown through drillers without prior land preparation and disturbing the soil where previous crop stubbles are present. Zero tillage not only reduce the cost of cultivation it also reduces the soil erosion, crop duration and irrigation requirement and weed effect which is better than tillage. Zero Tillage (ZT) also called No Tillage or Nil Tillage.
 

Statement 2: Direct Seeded Rice Zero-Tillage DSR is an alternative crop establishment method for rice where seeds are sown directly without raising them in a nursery, and can be done in zero-tillage conditions.
 

Statement 3: If crops are not burnt and land is prepared through mechanical agitation, carbon sequestration remains possible.

18. According to India’s National Policy on Biofuels, which of the following can be used as raw materials for the production of biofuels?
 

1. Cassava

2. Damaged wheat grains

3. Groundnut seeds

4. Horse gram

5. Rotten potatoes

6. Sugar beet
 

Select the correct answer using the code given below:
 

(a) 1,2,5 and 6 only

(b) 1,3,4 and 6 only

(c)2,3,4 and 5 only

(d) 1,2,3,4,5 and 6.
 

Answer: (a)

From the policy document: “The scope of the Policy encompasses following categories of fuels as “Biofuels” which can be used as transportation fuel or in stationery applications:—

  1. ‘bioethanol’: ethanol produced from biomass such as sugar containing materials, like sugar cane, sugar beet, sweet sorghum etc.; starch containing materials such as corn, cassava, rotten potatoes, algae etc.; and, cellulosic materials such as bagasse, wood waste, agricultural and forestry residues or other renewable resources like industrial waste;

  2. ‘biodiesel’: a methyl or ethyl ester of fatty acids produced from non-edible vegetable oils, acid oil, used cooking oil or animal fat and bio-oil;

See http://petroleum.nic.in/sites/default/files/biofuelpolicy2018_1.pdf

19. Which one of the following statements best describes the term ‘Social Cost of Carbon’?
It is a measure, in monetary value, of the

(a) long-term damage done by a tonne of CO2 emissions in a given year.

(b) requirement of fossil fuels for a country to provide goods and services to its citizens, based on the burning of those fuels.

(c)efforts put in by a climate refugee to adapt to live in a new place.

(d) contribution of an individual person to the carbon footprint on the planet Earth.
 

Answer: (a)

The social cost of carbon (SCC) is the marginal cost of the impacts caused by emitting one extra tonne of greenhouse gas (carbon dioxide equivalent) at any point in time, inclusive of ‘non-market’ impacts on the environment and human health. The purpose of putting a price on a ton of emitted CO2 is to aid policymakers or other legislators in evaluating whether a policy designed to curb climate change is justified.

20. In the context of India, which of the following is/are considered to be practice(s) of eco-friendly agriculture?
 

1. Crop diversification

2. Legume intensification

3. Tensiometer use

4. Vertical farming
 

Which of the statements given above is/are correct ?
 

(a) 1,2 and 3 only

(b) 3 only

(c)4 only

(d) 1,2,3 and 4
 

Answer: (d)
All these practices in agriculture such as Crop diversification, Legume intensification, Tensiometer use, Vertical farming, etc are eco-friendly. Hence the correct answer is 1, 2, 3, and 4.

21. Consider the following minerals:

1. Bentonite

2. Chromite

3. Kyanite

4. Sillimanite
 

In India, which of the above is/are officially designated as major minerals?
 

(a) 1 and 2 only

(b) 4 only

(c) 1 and 3 only

(d) 2, 3 and 4 only
 

Answer: (d)

As per Govt. of India Notification S.O. 423(E) dated 10th February 2015 Bentonite has been declared as 'Minor Mineral'. Hence statement 1 is not correct. 

Important Points -

The Central Government has declared the following minerals as minor minerals: 

Boulder, shingle, chalcedony pebbles used for ball mill purposes only, lime shell, kankar. And limestone used in kilns for the manufacture of lime used as a building material. Murrum, brick-earth, fuller’s earth, bentonite, road metal, reh-matti. 

Slate and shale when used for building material, marble, stone used for making household utensils. Quartzite and sandstone when used for purposes of building or for making road metal and household utensils, saltpeter, and ordinary earth used or filling or leveling purposes in construction or embankments, roads, railways, building. 

Additional Information 

Bentonite is essentially a highly plastic clay containing not less than 85% clay mineral, montmorillonite. 

Chromite (Cr) is the single commercially viable ore of chromium which is chemically known as iron chromium oxide (Fe.Cr2O4). 

Kyanite, Sillimanite, and andalusite are anhydrous aluminosilicate minerals that have the same chemical formula Al2O3 but differ in the crystal structure and physical properties.

22. With reference to Ocean Mean Temperature(OMT),which of the following statements is/are correct?
1. OMT is measured upto a depth of 26 degree Celsius isotherm which is 129 meters in the south-western Indian Ocean during January-March.

2. OMT collected during January-March can be used in assessing whether the amount of rainfall in monsoon will be less or more than a certain long-term mean.
Select the correct answer using the code given below:
 

(a) 1 only

(b) 2 only

(c)Both 1 and 2

(d) Neither 1 nor 2
 

Answer: (b)

Statement 1 and Statement 2: The depth for the 26 degree isotherm is 50-100 metres, not 129 metres as the question suggests.

Ocean mean temperature is a better indicator of monsoon rainfall than sea surface temperatures(SST). This is because OMT measures the ocean’s thermal energy, and on the other hand SST is influenced by surface winds, evaporation, thick clouds etc. OMT is a more stable indicator.

23. With reference to India’s Desert National Park, which of the following statements are correct?
 

1. It is spread over two districts.

2. There is no human habitation inside the Park.

3. It is one of the natural habitats of Great Indian Bustard.
 

Select the correct answer using the code given below:
 

(a) 1 and 2 only

(b) 2 and 3 only

(c) 1 and 3 only

(d) 1,2 and 3
 

Answer: (c)

Statement 1: Situated in Jaisalmer and Barmer districts of Indian state Rajasthan.

Statement 2: This is clearly incorrect. A national park can have human habitation.

Statement 3: This is common knowledge.

24. Consider the following statements:

1. 36% of India’s districts are classified as “overexploited” or “critical” by the Central Ground Water Authority(CGWA).

2. CGWA was formed under the Environment (Protection) Act.

3. India has the largest area under groundwater irrigation in the world.

Which of the statements given above is/are correct ?

(a) 1 only

(b) 2 and 3 only

(c) 2 only

(d) 1 and 3 only

Answer: (b)

Out of the 5723 assessment units assessed jointly by State Ground Water Departments and CGWB in the country, as per the report of Niti Aayog, the following is the data - Safe-71%, Semi-critical-10%, Critical-4%, and Over Exploited-15%. Hence statement 1 is not correct. 

Central Ground Water Authority (CGWA) was constituted under Section 3(3) of the Environment (Protection) Act, (1986). Hence statement 2 is correct. 

It regulates the extraction of groundwater through guidelines that are updated regularly. At 39 million hectares (67% of its total irrigation), India has the world’s largest groundwater well-equipped irrigation system. Hence statement 3 is correct. 

China with 19 mha is second, the USA with 17 mha is third. Pakistan and Bangladesh are at fourth and fifth respectively in largest groundwater reserve. 


Important Info : Central Ground Water Board Central Ground Water Board was established in 1970 by renaming the Exploratory Tube wells Organization Central Ground Water Board (CGWB), a subordinate office of the Ministry of Water Resources, Government of India. It is the National Apex Agency entrusted with the responsibility of providing scientific inputs for the management of groundwater, exploration, monitoring, assessment, augmentation, and regulation of groundwater resources of the country. CGWB is under the Ministry of Jal Shakti, Department of Water Resources, River Development and Ganga Rejuvenation.

25. Among the following Tiger Reserves, which one has the largest area under “Critical Tiger Habitat” ?
 

(a) Corbett

(b) Ranthambore

(c) Nagarjunasagar- Srisailam

(d) Sunderbans
 

Answer: (c)

Nagarjunasagar- Srisailam is the largest tiger reserve forest in India and a critical tiger habitat. https://etelangana.org/uploads/ebooks/Telangana_Article__2014-04-02_051708/2014-04-08_065702_Chenchu_Tiger_reserve.pdf

bottom of page